Med Surg Neuro Superset

Ace your homework & exams now with Quizwiz!

A patient is being treated for increased intracranial pressure. Which activities below should the patient avoid performing?* A. Coughing B. Sneezing C. Talking D. Valsalva maneuver E. Vomiting F. Keeping the head of the bed between 30- 35 degrees

A. Coughing B. Sneezing D. Valsalva maneuver E. Vomiting The answers are A, B, D, and E. These activities can increase ICP.

Select all the signs and symptoms that occur with increased ICP:* A. Decorticate posturing B. Tachycardia C. Decrease in pulse pressure D. Cheyne-stokes E. Hemiplegia F. Decerebrate posturing

A. Decorticate posturing D. Cheyne-stokes E. Hemiplegia F. Decerebrate posturing The answers are A, D, E, and F. Option B is wrong because bradycardia (not tachycardia) happens in the late stage along with an INCREASE (not decrease) in pulse pressure.

a nurse is caring for a client who has a left intracranial hemorrhage from a stroke. which of the following findings should the nurse expect a. spasticity of the left foot b. negative Babinski reflex c. ocular hypertension d. right-sided hemiplegia

. right-sided hemiplegia

38. A patient with spinal cord injury is ready to be discharged home. A family member asks the nurse to review potential complications one more time. What are the potential complications that should be monitored for in this patient? Select all that apply. A) Orthostatic hypotension B) Autonomic dysreflexia C) DVT D) Salt-wasting syndrome E) Increased ICP

A, B, C Feedback: For a spinal cord-injured patient, based on the assessment data, potential complications that may develop include DVT, orthostatic hypotension, and autonomic dysreflexia. Salt-wasting syndrome or increased ICP are not typical complications following the immediate recovery period.

30. The nurse is admitting a patient to the unit who is scheduled for removal of an intracranial mass. What diagnostic procedures might be included in this patient's admission orders? Select all that apply. A) Transcranial Doppler flow study B) Cerebral angiography C) MRI D) Cranial radiography E) Electromyelography (EMG)

A, B, C Feedback: Preoperative diagnostic procedures may include a CT scan to demonstrate the lesion and show the degree of surrounding brain edema, the ventricular size, and the displacement. An MRI scan provides information similar to that of a CT scan with improved tissue contrast, resolution, and anatomic definition. Cerebral angiography may be used to study a tumor's blood supply or to obtain information about vascular lesions. Transcranial Doppler flow studies are used to evaluate the blood flow within intracranial blood vessels. Regular x-rays of the skull would not be diagnostic for an intracranial mass. An EMG would not be ordered prior to intracranial surgery to remove a mass.

36. A family member of a patient diagnosed with Huntington disease calls you at the clinic. She is requesting help from the Huntington's Disease Society of America. What kind of help can this patient and family receive from this organization? Select all that apply. A) Information about this disease B) Referrals C) Public education D) Individual assessments E) Appraisals of research studies

A, B, C Feedback: The Huntington's Disease Society of America helps patients and families by providing information, referrals, family and public education, and support for research. It does not provide individual assessments or appraisals of individual research studies.

40. An adult patient has sought care for the treatment of headaches that have become increasingly severe and frequent over the past several months. Which of the following questions addresses potential etiological factors? Select all that apply? A) "Are you exposed to any toxins or chemicals at work?" B) "How would you describe your ability to cope with stress?" C) "What medications are you currently taking?" D) "When was the last time you were hospitalized?" E) "Does anyone else in your family struggle with headaches?"

A, B, C, E Feedback: Headaches are multifactorial, and may involve medications, exposure to toxins, family history, and stress. Hospitalization is an unlikely contributor to headaches.

The patient has a blood pressure of 130/88 and ICP reading of 12. What is the patient's cerebral perfusion pressure, and how do you interpret this as the nurse?* A. 90 mmHg, normal B. 62 mmHg, abnormal C. 36 mmHg, abnormal D. 56 mmHg, normal

A. 90 mmHg, normal The answer is A. CPP is calculated by the following formula: CPP=MAP-ICP. The patient's CPP is 90 and this is normal. A normal CPP is 60-100 mmHg.

1. Select the main structures below that play a role with altering intracranial pressure:* A. Brain B. Neurons C. Cerebrospinal Fluid D. Blood E. Periosteum F. Dura mater

A. Brain C. Cerebrospinal Fluid D. Blood The answers are A, C, and D. Inside the skull are three structures that can alter intracranial pressure. They are the brain, cerebrospinal fluid (CSF), and blood.

A client is admitted with a brain abscess. Which diagnostic assessment intervention does the nurse question as nonspecific to the diagnosis?

Bone scan A bone scan is done to determine new areas of bone growth, areas of metastatic lesions, and osteoporosis. An EEG is obtained to localize the lesion. Throat, ear, nose, and blood (aerobic and anaerobic) cultures are done to determine the primary source of infection. Sinus and mastoid x-rays are requested to see if they are the primary source of the infection causing the brain abscess.

A client has been admitted with a diagnosis of stroke (brain attack). The nurse suspects that the client has had a right hemisphere stroke because the client exhibits which symptoms?

Impulsiveness and smiling Impulsiveness and smiling are symptoms indicative of a right hemisphere stroke. Aphasia, cautiousness, the inability to discriminate words, quick to anger, and frustration are symptoms indicative of a left hemisphere stroke.

A nurse is assessing a client who has a new diagnosis of mastoiditis. What manifestation should the nurse expect? a. swelling behind the affected ear b. facial drooping on the affected side c. nystagmus on the affected side d. pearly gray color of the affected eardrum

a. swelling behind the affected ear Mastoiditis refers to an inflammation of the temporal bone behind the ear. Manifestations of mastoiditis include swelling and pain behind the ear.

a nurse is assessing a client who is unconscious. the client has a rhythmical breathing pattern of a rapid deep respirations followed by rapid shallow respirations, alternating with period of apnea. the nurse should document that the client is experiencing which of the following types of respirations a. orthopea b. cheyne-stokes c. paradoxical -chest wall contracts during inspiration and expand during expiration d. Kussmaul: deep, rapid respiration

cheyne-stokes

a nurse in an emergency department is assessing a client who sustained a fall off of a roof. which of the following findings should the nurse identify as an indication of a basilar skull fracture a. depressed fracture of the forehead b. clear fluid coming from the nares c. motor loss on one side of the body d. bleeding from the top of the scalp

clear fluid coming from the nares

A nurse is caring for a client who has a traumatic brain injury and assumes a decerebrate posture in response to noxious stimuli. What reaction should the nurse anticipate when drawing a blood sample? a. rigidly extends arms b. internally flexes wrists c. curls into fetal position d. internally rotates his legs

a. rigidly extends arms A client who exhibits a decerebrate posture rigidly extends and pronates his four extremities and externally rotates his wrists. Decerebrate posturing indicates severe brain stem injury and late neurologic decline.

a nurse is caring for a client who has encephalitis due to the west Nile virus. which of the following actions should the nurse take a. place the client in respiratory isolation b. monitor vital signs every 2 hr c. assess neurological status every 4 hr d. maintain the client in a modified Trendelenburg position e. keep the client's room darkened

monitor vital signs every 2 hr c. assess neurological status every 4 hr keep the client's room darkened

a nurse is assessing a client who has cataracts. which of the following findings should the nurse expect a. pupils nonreactive to light b. opacity visible behind the pupil c. white circle around the outside border of the iris d. increased intraocular pressure

opacity visible behind the pupil

a nurse in an acute care facility is preparing to admit a client who has myasthenia gravis. which of the following supplies should the nurse place at the client's bedside

oral -nasal suction equipment

a nurse is assessing a client who has a new diagnosis of acute angle-closure glaucoma. the nurse should anticipate the client to report which of the following manifestations a. multiple floaters b. flashes of light in front of the eye c. severe eye pain d. double vision

severe eye pain halos around the light blurred vision brow pain, headache, N/V

a nurse is providing teaching to a client who is scheduled for an electroencephalogram (EEG) in the morning. which of the following pieces of information should the nurse share

shampoo your hair before the procedure and don't use any styling product afterward

a nurse is assessing a client who is postoperative following a craniotomy and has a urine output of 600 mL/hr. the nurse suspects the client has manifestations of diabetes insipidus. which of the following laboratory values should the nurse plan to obtain to assess for DI a. blood urea nitrogen b. blood glucose c. urine ketones d. specific gravity

specific gravity

a nurse is assessing a client who has a new diagnosis of mastoiditis. which of the following manifestation should the nurse expect

swelling behind the affected ear

a nurse is reviewing the medical history of a client who is scheduled for a magnetic resources imaging (MRI) examination of the cervical vertebra. which of the following pieces of information in the client's history is a contraindication to this procedure a. the client has a new tattoo b. the client is unable to sit upright. c. the client has a history of peripheral vascular disease d. the client has a pacemaker

the client has a pacemaker

a nurse is caring for a client who has a traumatic brain injury and assumes a decerebrate posture in response to noxious stimuli. which of the following reactions should the nurse anticipate when drawing a blood sample? a. the client rigidly extends his arms b. the client internally flexes his wrists c. the client curls into a fetal position d. the client internally rotates his legs

the client rigidly extends his arms

a nurse is caring for a client who is recovering from resent stroke. which of the following assessments is the nurse's priority

the client's ability to clear oral secretions to reduce aspiration

the nurse is assessing a client with a closed head injury who has received mannitol for manifestations of increased intracranial pressure (ICP). which of the following findings indicates that the medication is having a therapeutic effect

the client's serum osmolarity is 310 mOsm/L

a nurse is providing teaching to a client who has a history of tonic-clonic seizures and is scheduled for a standard electroencephalogram (EEG). which of the following instructions should the nurse include in the teaching

thoroughly shampoo her hair prior to the EEG

a nurse is walking along the unit when she sees smoke coming from the central supply room. after activating the fire alarm, which of the following actions should the nurse take

turn off sources of oxygen near the fire place wet towels at the base of the fire

a nurse is caring for a client who begins to have a generalized tonic-clonic seizure while lying in bed. which of the following actions should the nurse take a. insert an oral airway b. turn the client onto a side c. restrict movement of the client's limbs d. place a pillow under the client's head

turn the client onto a side

a nurse is providing teaching to the family of a client who has a new diagnosis of amyotrophic lateral sclerosis (ALS). which of the following findings is an early manifestation of ALS a. sensory dysfunction b. weakness of the distal extremities c. decreased vision d. altered temperature regulation

weakness of the distal extremities

a nurse is assessing a client who has Guillain-barre (ascending weakness and paralysis) syndrome. which of the following findings should the nurse expects a. tonic-closure seizures b. report of a severe headache c. weakness of the lower extremities d. decreased level of consciousness

weakness of the lower extremities

a nurse is assessing a client who sustained a recent head injury. which of the following findings should the nurse recognize as a manifestation of increased intracranial pressure a. widened pulse pressure b. tachycardia c. periorbital edema d. decrease in urine output

widened pulse pressure

a nurse is providing preoperative teaching for a client who will undergo laser assisted in situ keratomleusis (LASIK) surgery which of the following pieces of information should the nurse include

you might need your glasses after the surgery

a nurse is providing discharge teaching to a client who is postoperative following scleral buckling to repair a detached retina. which of the following instructions should the nurse include in the teaching

you should avoid reading for 1 week

a nurse is preparing a client for an electroencephalogram (EEG.) which of the following pieces of information should the nurse share with the client

you will begin by lying still with your eyes close -takes 45min-2hr

a nurse is teaching a client about computed tomography (CT) scanning of the brain which of the following teaching points should the nurse include

you will have to lie very still on a long, narrow table during the test takes 5 min client lie supine

a nurse is caring for a client who has receptive aphasia which of the following communication problems should the nurse expect when assessing the client a. the client can not name object or formulate sentences or phrases--expressive aphasia b. the client can not articulate correctly due to muscle weakness of the mouth and tongue--dysarthria The c.client is unable to understand words or sentences she hears d. the client speaks words that substitute for those she intends to say--apraxia

client is unable to understand words or sentences she hears

a nurse is caring for a client who has Meniere's disease. the nurse should identify that Meniere's disease affects which structure of the ear. cochlea: inner ear problem a. eustachian tube-infection of the middle ear b. cochlea c. perichondrium d. eardrum

cochlea

a nurse is caring for a client who has had repeated middle ear infections. the client reports that the provider said the infection is due to an obstruction of the structure that connects the middle ear to the throat. the nurse should identify that the provider was referring to which of the following structure

eustachian tube

a nurse is caring for a client who had a cerebrovascular accident. the client appears alert and engage during a visit but does not respond verbally to questions. the nurse should document this as which of the following alterations a. expressive aphasia b. dysarthria c. receptive aphasia d. dysphagia

expressive aphasia

a nurse is caring for a client who has a hearing impairment. which of the following actions should the nurse take when communicating with the client

face the client when speaking

a nurse is teaching a class of new parent about otitis media. which of the following manifestations should the nurse include in the teaching? a. high-pitched sound heard in the ear b. intermitted rapid eye movement c. itching of the external canal d. feelings of fullness in the ear

feelings of fullness in the ear

a nurse is caring for a client who has expressive aphasia following a stroke. the nurse should identify that stroke affect which of the following lobes of the client's brain a. occipital- vision b. temporal-understanding speech c. frontal-verbal expression of thoughts d. limbic-memory and learning

frontal-verbal expression of thoughts

a nurse is preparing to test the function of cranial nerve X. which of the following assessment procedures should the nurse use? a. have the client open his mouth and say "aah" b. ask the client to identify the scent of a coffee c. use a tongue blade to provoke a gag reflex d. have the client smile and raise his eyebrows

have the client open his mouth and say "aah"

a nurse is caring for a client who has a cerebral lesion and develops hyperthermia. which of the following areas of the client brain is affected. a. wernicke's area-- language and speech comprehension b. cerebral cortex--thought process and higher function of brain c. basal ganglia- motor control and learning d. hypothalamus

hypothalamus

a nurse is providing discharge teaching to a client who is postoperative following cataract surgery and has an intraocular lens implant. which of the following statements by the client indicates understanding of the instruction

i will avoid bending over

a nurse names 3 object for a client to remember, ask the client to repeat them, and tells the client he will have to repeat them again in a few minutes. after 5 min, the nurse asks the client to name the object. the nurse is using this strategy to test which type of memory a. remote b. sensory c. immediate d. recall

immediate

a nurse in an emergency department has assessed a client's airway, breathing, and circulation following a head injury from a fall at work. which of the following actions is the priority for the nurse to perform next?

immobilize the client's cervical spine

a nurse is assessing a client who recently experienced a head injury. which of the findings should the nurse identify as an indication of short-term memory impairment a. inability to remember current age b. inability to count backward c. inability to locate eyeglasses d. inability to recall names of family members

inability to locate eyeglasses

a nurse is assessing a client who has increased intracranial pressure and has received intravenous mannitol. which of the following findings indicates a therapeutic effect of this medication.

increased urine output

a nurse in a clinic is providing teaching to an adolescent client who has recurrent external otitis. which of the following instructions should the nurse include in the teaching

instill a diluted alcohol solution into the ear after swimming

a charge nurse is observing a newly licensed nurse irrigate a client's ear, which is impacted with cerumen. which of the following actions requires the charge nurse to intervene

instilling 50 mL of fluid with each irrigation instead of 5-10ml

a nurse is assessing a client who was admitted to the facility for observation following a closed head injury. which of the following is the priority assessment the nurse should perform to determine a change in the client's neurological status a. vital sign b. body posture c. level of consciousness d. examination of pupils

level of consciousness

a nurse is providing teaching to the family of a client who has stage II Alzheimer's disease. which of the following pieces of information should the nurse include in the teaching

limit choices offered to the client

a nurse is planning care for a client following a stroke. which of the following interventions should the nurse identify as the priority in the client's plan of care

monitor the client for increased intracranial pressure

a nurse is teaching a client who has a new diagnosis of primary open-angle glaucoma (POAG). which of the following pieces of information should the nurse include in the teaching

-driving can be dangerous due to the loss of peripheral vision -laser surgery can help reestablish the flow of aqueous humor

A client is considering treatments for a malignant brain tumor. Which statement by the client indicates a need for further instruction by the nurse?

"Antibiotics will help minimize the size of the tumor." Antibiotics are used to treat a brain abscess, not a malignant tumor. Chemotherapy, radiation, and surgery are often used in conjunction with each other to treat malignancies. For a craniotomy, several burr holes are drilled into the skull, and a saw is used to remove a piece of bone (bone flap) to expose the tumor area. The goals of treatment of brain tumor are to decrease tumor size, improve quality of life, and improve survival time.

The daughter of a client who has had a stroke asks the nurse for additional resources. What is the nurse's best response?

"Go to the National Stroke Association website." The National Stroke Association is a specific and reliable resource that can be recommended. Hospice applies only to the client who will be requiring palliative end-of-life care. The Internet is too broad; unless the nurse recommends a specific website, the client's daughter may not find quality information. The nurse caring for the client is responsible for obtaining information that is readily available or for procuring a request from the health care provider for a consultation with the social worker.

The nurse is teaching a client and family about home care after a stroke. Which statement made by the client's spouse indicates a need for further teaching?

"I should spend all my time with my husband in case I'm needed." Family members can start to feel socially isolated when caring for a loved one. The family may need to plan for regular respite care in a structured day-care respite program or through relief provided by a friend or neighbor. The life changes associated with stroke often cause a change in the client's self-esteem. The client who has had a stroke should maintain a regular medication regimen, such as anticoagulant therapy, to prevent another stroke. Once the home health nurse has assessed the home environment, he or she will notify the health care provider of the need for ancillary services, such as a physical therapist. The physical therapist will identify adaptive equipment needs, will request them, and then will instruct the client about their use, along with developing an exercise program.

A client is eating a soft diet while recovering from a stroke. The client reports food accumulating in the cheek of the affected side. What is the nurse's best response?

"Let's see if the speech-language pathologist can help." The speech-language pathologist identifies strategies to prevent food from accumulating in the cheek of the affected side of a client recovering from a stroke. The correct technique to improve swallowing is the chin-tuck method; however, the speech pathologist will assist the client with tongue exercises that will help move the food bolus to the unaffected side. Solid food is not appropriate for the client with chewing and swallowing challenges. The dietitian consults with the health care team if the client has had weight loss problems, or if abnormal laboratory results indicate a nutritional deficit.

The nurse is teaching the spouse and client who has had a brain attack about rehabilitation. Which statement by the spouse demonstrates understanding of the nurse's instruction?

"The rehabilitation therapist will help identify changes needed at home." The rehabilitation therapist and home health professionals assist the client and family in adapting the home environment to the client's needs and assess the client's need for therapy. Any medication regimen for the client must be maintained. Rehabilitation is much more comprehensive than physical therapy. The family should develop a home routine that provides structure, repetition, and consistency.

A client is being discharged home after surgery for brain cancer. Which statement by the client's spouse indicates a correct understanding of the nurse's discharge teaching?

"We can find a support group through the local American Cancer Society." The American Cancer Society is a good community resource for clients with malignant tumors and their families. It is not a requirement that the client's spouse quit his or her job; resources are available for in-home client care. Although life need not be completely altered, it will never revert back to normal, as it was before the client's cancer occurred. The case manager helps coordinate care and teaching, but does not provide home care.

26. The nurse is planning the care of a patient who has been recently diagnosed with a cerebellar tumor. Due to the location of this patient's tumor, the nurse should implement measures to prevent what complication? Falls B) Audio hallucinations C) Respiratory depression D) Labile BP

A Feedback: A cerebellar tumor causes dizziness, an ataxic or staggering gait with a tendency to fall toward the side of the lesion, and marked muscle incoordination. Because of this, the patient faces a high risk of falls. Hallucinations and unstable vital signs are not closely associated with cerebellar tumors.

14. A patient with a C5 spinal cord injury is tetraplegic. After being moved out of the ICU, the patient complains of a severe throbbing headache. What should the nurse do first? A) Check the patient's indwelling urinary catheter for kinks to ensure patency. B) Lower the HOB to improve perfusion. C) Administer analgesia. D) Reassure the patient that headaches are expected after spinal cord injuries.

A Feedback: A severe throbbing headache is a common symptom of autonomic dysreflexia, which occurs after injuries to the spinal cord above T6. The syndrome is usually brought on by sympathetic stimulation, such as bowel and bladder distention. Lowering the HOB can increase ICP. Before administering analgesia, the nurse should check the patient's catheter, record vital signs, and perform an abdominal assessment. A severe throbbing headache is a dangerous symptom in this patient and is not expected.

1. A nurse is assessing a patient with an acoustic neuroma who has been recently admitted to an oncology unit. What symptoms is the nurse likely to find during the initial assessment? A) Loss of hearing, tinnitus, and vertigo B) Loss of vision, change in mental status, and hyperthermia C) Loss of hearing, increased sodium retention, and hypertension D) Loss of vision, headache, and tachycardia

A Feedback: An acoustic neuroma is a tumor of the eighth cranial nerve, the cranial nerve most responsible for hearing and balance. The patient with an acoustic neuroma usually experiences loss of hearing, tinnitus, and episodes of vertigo and staggering gait. Acoustic neuromas do not cause loss of vision, increased sodium retention, or tachycardia.

22. A patient who has experienced an ischemic stroke has been admitted to the medical unit. The patient's family in adamant that she remain on bed rest to hasten her recovery and to conserve energy. What principle of care should inform the nurse's response to the family? A) The patient should mobilize as soon as she is physically able. B) To prevent contractures and muscle atrophy, bed rest should not exceed 4 weeks. C) The patient should remain on bed rest until she expresses a desire to mobilize. D) Lack of mobility will greatly increase the patient's risk of stroke recurrence.

A Feedback: As soon as possible, the patient is assisted out of bed and an active rehabilitation program is started. Delaying mobility causes complications, but not necessarily stroke recurrence. Mobility should not be withheld until the patient initiates.

23. The nurse is caring for a patient whose recent health history includes an altered LOC. What should be the nurse's first action when assessing this patient? A) Assessing the patient's verbal response B) Assessing the patient's ability to follow complex commands C) Assessing the patient's judgment D) Assessing the patient's response to pain

A Feedback: Assessment of the patient with an altered LOC often starts with assessing the verbal response through determining the patient's orientation to time, person, and place. In most cases, this assessment will precede each of the other listed assessments, even though each may be indicated.

29. The school nurse has been called to the football field where player is immobile on the field after landing awkwardly on his head during a play. While awaiting an ambulance, what action should the nurse perform? A) Ensure that the player is not moved. B) Obtain the player's vital signs, if possible. C) Perform a rapid assessment of the player's range of motion. D) Assess the player's reflexes.

A Feedback: At the scene of the injury, the patient must be immobilized on a spinal (back) board, with the head and neck maintained in a neutral position, to prevent an incomplete injury from becoming complete. This is a priority over determining the patient's vital signs. It would be inappropriate to test ROM or reflexes.

30. The nurse is caring for a patient whose spinal cord injury has caused recent muscle spasticity. What medication should the nurse expect to be ordered to control this? A) Baclofen (Lioresal) B) Dexamethasone (Decadron) C) Mannitol (Osmitrol) D) Phenobarbital (Luminal)

A Feedback: Baclofen is classified as an antispasmodic agent in the treatment of muscles spasms related to spinal cord injury. Decadron is an anti-inflammatory medication used to decrease inflammation in both SCI and head injury. Mannitol is used to decrease cerebral edema in patients with head injury. Phenobarbital is an anticonvulsant that is used in the treatment of seizure activity.

37. A patient who is being treated in the hospital for a spinal cord injury is advocating for the removal of his urinary catheter, stating that he wants to try to resume normal elimination. What principle should guide the care team's decision regarding this intervention? A) Urinary retention can have serious consequences in patients with SCIs. B) Urinary function is permanently lost following an SCI. C) Urinary catheters should not remain in place for more than 7 days. D) Overuse of urinary catheters can exacerbate nerve damage.

A Feedback: Bladder distention, a major cause of autonomic dysreflexia, can also cause trauma. For this reason, removal of a urinary catheter must be considered with caution. Extended use of urinary catheterization is often necessary following SCI. The effect of a spinal cord lesion on urinary function depends on the level of the injury. Catheter use does not cause nerve damage, although it is a major risk factor for UTIs.

22. A patient is admitted to the neurologic ICU with a suspected diffuse axonal injury. What would be the primary neuroimaging diagnostic tool used on this patient to evaluate the brain structure? A) MRI B) PET scan C) X-ray D) Ultrasound

A Feedback: CT and MRI scans, the primary neuroimaging diagnostic tools, are useful in evaluating the brain structure. Ultrasound would not show the brain nor would an x-ray. A PET scan shows brain function, not brain structure.

17. A patient has developed diabetes insipidus after having increased ICP following head trauma. What nursing assessment best addresses this complication? A) Vigilant monitoring of fluid balance B) Continuous BP monitoring C) Serial arterial blood gases (ABGs) D) Monitoring of the patient's airway for patency

A Feedback: Diabetes insipidus requires fluid and electrolyte replacement, along with the administration of vasopressin, to replace and slow the urine output. Because of these alterations in fluid balance, careful monitoring is necessary. None of the other listed assessments directly addresses the major manifestations of diabetes insipidus.

19. A patient with Huntington disease has just been admitted to a long-term care facility. The charge nurse is creating a care plan for this patient. Nutritional management for a patient with Huntington disease should be informed by what principle? A) The patient is likely to have an increased appetite. B) The patient is likely to required enzyme supplements. C) The patient will likely require a clear liquid diet. D) The patient will benefit from a low-protein diet.

A Feedback: Due to the continuous involuntary movements, patients will have a ravenous appetite. Despite this ravenous appetite, patients usually become emaciated and exhausted. As the disease progresses, patients experience difficulty in swallowing and thin liquids should be avoided. Protein will not be limited with this disease. Enzyme supplements are not normally required.

17. A patient with a T2 injury is in spinal shock. The nurse will expect to observe what assessment finding? A) Absence of reflexes along with flaccid extremities B) Positive Babinski's reflex along with spastic extremities C) Hyperreflexia along with spastic extremities D) Spasticity of all four extremities

A Feedback: During the period immediately following a spinal cord injury, spinal shock occurs. In spinal shock, all reflexes are absent and the extremities are flaccid. When spinal shock subsides, the patient demonstrates a positive Babinski's reflex, hyperreflexia, and spasticity of all four extremities.

14. The nurse has created a plan of care for a patient who is at risk for increased ICP. The patient's care plan should specify monitoring for what early sign of increased ICP? A) Disorientation and restlessness B) Decreased pulse and respirations C) Projectile vomiting D) Loss of corneal reflex

A Feedback: Early indicators of ICP include disorientation and restlessness. Later signs include decreased pulse and respirations, projectile vomiting, and loss of brain stem reflexes, such as the corneal reflex.

38. When preparing to discharge a patient home, the nurse has met with the family and warned them that the patient may exhibit unexpected emotional responses. The nurse should teach the family that these responses are typically a result of what cause? A) Frustration around changes in function and communication B) Unmet physiologic needs C) Changes in brain activity during sleep and wakefulness D) Temporary changes in metabolism

A Feedback: Emotional problems associated with stroke are often related to the new challenges around ADLs and communication. These challenges are more likely than metabolic changes, unmet physiologic needs, or changes in brain activity, each of which should be ruled out.

10. The nurse is assessing a patient with a suspected stroke. What assessment finding is most suggestive of a stroke? A) Facial droop B) Dysrhythmias C) Periorbital edema D) Projectile vomiting

A Feedback: Facial drooping or asymmetry is a classic abnormal finding on a physical assessment that may be associated with a stroke. Facial edema is not suggestive of a stroke and patients less commonly experience dysrhythmias or vomiting.

23. A gerontologic nurse is advocating for diagnostic testing of an 81-year-old patient who is experiencing personality changes. The nurse is aware of what factor that is known to affect the diagnosis and treatment of brain tumors in older adults? A) The effects of brain tumors are often attributed to the cognitive effects of aging. B) Brain tumors in older adults do not normally produce focal effects. C) Older adults typically have numerous benign brain tumors by the eighth decade of life. D) Brain tumors cannot normally be treated in patient over age 75.

A Feedback: In older adult patients, early signs and symptoms of intracranial tumors can be easily overlooked or incorrectly attributed to cognitive and neurologic changes associated with normal aging. Brain tumors are not normally benign and they produce focal effects in all patients. Treatment options are not dependent primarily on age.

26. The nurse is caring for a patient who sustained a moderate head injury following a bicycle accident. The nurse's most recent assessment reveals that the patient's respiratory effort has increased. What is the nurse's most appropriate response? A) Inform the care team and assess for further signs of possible increased ICP. B) Administer bronchodilators as ordered and monitor the patient's LOC. C) Increase the patient's bed height and reassess in 30 minutes. D) Administer a bolus of normal saline as ordered.

A Feedback: Increased respiratory effort can be suggestive of increasing ICP, and the care team should be promptly informed. A bolus of IV fluid will not address the problem. Repositioning the patient and administering bronchodilators are insufficient responses, even though these actions may later be ordered.

34. After a major ischemic stroke, a possible complication is cerebral edema. Nursing care during the immediate recovery period from an ischemic stroke should include which of the following? A) Positioning to avoid hypoxia B) Maximizing PaCO2 C) Administering hypertonic IV solution D) Initiating early mobilization

A Feedback: Interventions during this period include measures to reduce ICP, such as administering an osmotic diuretic (e.g., mannitol), maintaining the partial pressure of carbon dioxide (PaCO2) within the range of 30 to 35 mm Hg, and positioning to avoid hypoxia. Hypertonic IV solutions are not used unless sodium depletion is evident. Mobilization would take place after the immediate threat of increased ICP has past.

25. A male patient presents at the free clinic with complaints of impotency. Upon physical examination, the nurse practitioner notes the presence of hypogonadism. What diagnosis should the nurse suspect? A) Prolactinoma B) Angioma C) Glioma D) Adrenocorticotropic hormone (ACTH)-producing adenoma

A Feedback: Male patients with prolactinomas may present with impotence and hypogonadism. An ACTH-producing adenoma would cause acromegaly. The scenario contains insufficient information to know if the tumor is an angioma, glioma, or neuroma.

30. An older adult has encouraged her husband to visit their primary care provider, stating that she is concerned that he may have Parkinson's disease. Which of the wife's descriptions of her husband's health and function is most suggestive of Parkinson's disease? A) "Lately he seems to move far more slowly than he ever has in the past." B) "He often complains that his joints are terribly stiff when he wakes up in the morning." C) "He's forgotten the names of some people that we've known for years." D) "He's losing weight even though he has a ravenous appetite."

A Feedback: Parkinson's disease is characterized by bradykinesia. It does not manifest as memory loss, increased appetite, or joint stiffness.

37. A patient with a new diagnosis of amyotrophic lateral sclerosis (ALS) is overwhelmed by his diagnosis and the known complications of the disease. How can the patient best make known his wishes for care as his disease progresses? A) Prepare an advance directive. B) Designate a most responsible physician (MRP) early in the course of the disease. C) Collaborate with representatives from the Amyotrophic Lateral Sclerosis Association. D) Ensure that witnesses are present when he provides instruction.

A Feedback: Patients with ALS are encouraged to complete an advance directive or "living will" to preserve their autonomy in decision making. None of the other listed actions constitutes a legally binding statement of end-of-life care.

18. A patient who suffered an ischemic stroke now has disturbed sensory perception. What principle should guide the nurse's care of this patient? A) The patient should be approached on the side where visual perception is intact. B) Attention to the affected side should be minimized in order to decrease anxiety. C) The patient should avoid turning in the direction of the defective visual field to minimize shoulder subluxation. D) The patient should be approached on the opposite side of where the visual perception is intact to promote recovery.

A Feedback: Patients with decreased field of vision should first be approached on the side where visual perception is intact. All visual stimuli should be placed on this side. The patient can and should be taught to turn the head in the direction of the defective visual field to compensate for this loss. The nurse should constantly remind the patient of the other side of the body and should later stand at a position that encourages the patient to move or turn to visualize who and what is in the room.

27. A patient has been admitted to the neurologic ICU with a diagnosis of a brain tumor. The patient is scheduled to have a tumor resection/removal in the morning. Which of the following assessment parameters should the nurse include in the initial assessment? A) Gag reflex B) Deep tendon reflexes C) Abdominal girth D) Hearing acuity

A Feedback: Preoperatively, the gag reflex and ability to swallow are evaluated. In patients with diminished gag response, care includes teaching the patient to direct food and fluids toward the unaffected side, having the patient sit upright to eat, offering a semisoft diet, and having suction readily available. Deep tendon reflexes, abdominal girth, and hearing acuity are less commonly affected by brain tumors and do not affect the risk for aspiration.

40. A nurse is planning discharge education for a patient who underwent a cervical diskectomy. What strategies would the nurse assess that would aid in planning discharge teaching? A) Care of the cervical collar B) Technique for performing neck ROM exercises C) Home assessment of ABGs D) Techniques for restoring nerve function

A Feedback: Prior to discharge, the nurse should assess the patient's use and care of the cervical collar. Neck ROM exercises would be contraindicated and ABGs cannot be assessed in the home. Nerve function is not compromised by a diskectomy.

4. The nurse is caring for a patient with increased intracranial pressure (ICP) caused by a traumatic brain injury. Which of the following clinical manifestations would suggest that the patient may be experiencing increased brain compression causing brain stem damage? A) Hyperthermia B) Tachycardia C) Hypertension D) Bradypnea

A Feedback: Signs of increasing ICP include slowing of the heart rate (bradycardia), increasing systolic BP, and widening pulse pressure. As brain compression increases, respirations become rapid, BP may decrease, and the pulse slows further. A rapid rise in body temperature is regarded as unfavorable. Hyperthermia increases the metabolic demands of the brain and may indicate brain stem damage.

33. A patient is postoperative day 1 following intracranial surgery. The nurse's assessment reveals that the patient's LOC is slightly decreased compared with the day of surgery. What is the nurse's best response to this assessment finding? A) Recognize that this may represent the peak of post-surgical cerebral edema. B) Alert the surgeon to the possibility of an intracranial hemorrhage. C) Understand that the surgery may have been unsuccessful. D) Recognize the need to refer the patient to the palliative care team.

A Feedback: Some degree of cerebral edema occurs after brain surgery; it tends to peak 24 to 36 hours after surgery, producing decreased responsiveness on the second postoperative day. As such, there is not necessarily any need to deem the surgery unsuccessful or to refer the patient to palliative care. A decrease in LOC is not evidence of an intracranial hemorrhage.

33. The nurse caring for a patient diagnosed with Parkinson's disease has prepared a plan of care that would include what goal? A) Promoting effective communication B) Controlling diarrhea C) Preventing cognitive decline D) Managing choreiform movements

A Feedback: The goals for the patient may include improving functional mobility, maintaining independence in ADLs, achieving adequate bowel elimination, attaining and maintaining acceptable nutritional status, achieving effective communication, and developing positive coping mechanisms. Constipation is more likely than diarrhea and cognition largely remains intact. Choreiform movements are related to Huntington disease.

9. The patient has been diagnosed with aphasia after suffering a stroke. What can the nurse do to best make the patient's atmosphere more conducive to communication? A) Provide a board of commonly used needs and phrases. B) Have the patient speak to loved ones on the phone daily. C) Help the patient complete his or her sentences. D) Speak in a loud and deliberate voice to the patient.

A Feedback: The inability to talk on the telephone or answer a question or exclusion from conversation causes anger, frustration, fear of the future, and hopelessness. A common pitfall is for the nurse or other health care team member to complete the thoughts or sentences of the patient. This should be avoided because it may cause the patient to feel more frustrated at not being allowed to speak and may deter efforts to practice putting thoughts together and completing a sentence. The patient may also benefit from a communication board, which has pictures of commonly requested needs and phrases. The board may be translated into several languages.

29. When caring for a patient with increased ICP the nurse knows the importance of monitoring for possible secondary complications, including syndrome of inappropriate antidiuretic hormone (SIADH). What nursing interventions would the nurse most likely initiate if the patient developed SIADH? A) Fluid restriction B) Transfusion of platelets C) Transfusion of fresh frozen plasma (FFP) D) Electrolyte restriction

A Feedback: The nurse also assesses for complications of increased ICP, including diabetes insipidus, and SIADH. SIADH requires fluid restriction and monitoring of serum electrolyte levels. Transfusions are unnecessary.

21. The nurse is caring for a patient with permanent neurologic impairments resulting from a traumatic head injury. When working with this patient and family, what mutual goal should be prioritized? A) Achieve as high a level of function as possible. B) Enhance the quantity of the patient's life. C) Teach the family proper care of the patient. D) Provide community assistance.

A Feedback: The overarching goals of care are to achieve as high a level of function as possible and to enhance the quality of life for the patient with neurologic impairment and his or her family. This goal encompasses family and community participation.

30. Following diagnostic testing, a patient has been admitted to the ICU and placed on cerebral aneurysm precautions. What nursing action should be included in patient's plan of care? A) Supervise the patient's activities of daily living closely. B) Initiate early ambulation to prevent complications of immobility. C) Provide a high-calorie, low-protein diet. D) Perform all of the patient's hygiene and feeding.

A Feedback: The patient is placed on immediate and absolute bed rest in a quiet, nonstressful environment, because activity, pain, and anxiety elevate BP, which increases the risk for bleeding. As such, independent ADLs and ambulation are contraindicated. There is no need for a high-calorie or low-protein diet.

You're maintaining an external ventricular drain. The ICP readings should be?* A. 5 to 15 mmHg B. 20 to 35 mmHg C. 60 to 100 mmHg D. 5 to 25 mmHg

A. 5 to 15 mmHg The answer is A. Normal ICP should be 5 to 15 mmHg.

31. A patient is recovering from intracranial surgery performed approximately 24 hours ago and is complaining of a headache that the patient rates at 8 on a 10-point pain scale. What nursing action is most appropriate? A) Administer morphine sulfate as ordered. B) Reposition the patient in a prone position. C) Apply a hot pack to the patient's scalp. D) Implement distraction techniques.

A Feedback: The patient usually has a headache after a craniotomy as a result of stretching and irritation of nerves in the scalp during surgery. Morphine sulfate may also be used in the management of postoperative pain in patients who have undergone a craniotomy. Prone positioning is contraindicated due to the consequent increase in ICP. Distraction would likely be inadequate to reduce pain and a hot pack may cause vasodilation and increased pain.

34. The nurse is providing health education to a patient who has a C6 spinal cord injury. The patient asks why autonomic dysreflexia is considered an emergency. What would be the nurse's best answer? A) "The sudden increase in BP can raise the ICP or rupture a cerebral blood vessel." B) "The suddenness of the onset of the syndrome tells us the body is struggling to maintain its normal state." C) "Autonomic dysreflexia causes permanent damage to delicate nerve fibers that are healing." D) "The sudden, severe headache increases muscle tone and can cause further nerve damage."

A Feedback: The sudden increase in BP may cause a rupture of one or more cerebral blood vessels or lead to increased ICP. Autonomic dysreflexia does not directly cause nerve damage.

4. A patient with suspected Parkinson's disease is initially being assessed by the nurse. When is the best time to assess for the presence of a tremor? A) When the patient is resting B) When the patient is ambulating C) When the patient is preparing his or her meal tray to eat D) When the patient is participating in occupational therapy

A Feedback: The tremor is present while the patient is at rest; it increases when the patient is walking, concentrating, or feeling anxious. Resting tremor characteristically disappears with purposeful movement, but is evident when the extremities are motionless. Consequently, the nurse should assess for the presence of a tremor when the patient is not performing deliberate actions.

16. A patient is brought by ambulance to the ED after suffering what the family thinks is a stroke. The nurse caring for this patient is aware that an absolute contraindication for thrombolytic therapy is what? A) Evidence of hemorrhagic stroke B) Blood pressure of ³ 180/110 mm Hg C) Evidence of stroke evolution D) Previous thrombolytic therapy within the past 12 months

A Feedback: Thrombolytic therapy would exacerbate a hemorrhagic stroke with potentially fatal consequences. Stroke evolution, high BP, or previous thrombolytic therapy does not contraindicate its safe and effective use.

37. A hospital patient has experienced a seizure. In the immediate recovery period, what action best protects the patient's safety? A) Place the patient in a side-lying position. B) Pad the patient's bed rails. C) Administer antianxiety medications as ordered. D) Reassure the patient and family members.

A Feedback: To prevent complications, the patient is placed in the side-lying position to facilitate drainage of oral secretions. Suctioning is performed, if needed, to maintain a patent airway and prevent aspiration. None of the other listed actions promotes safety during the immediate recovery period.

24. An 82-year-old man is admitted for observation after a fall. Due to his age, the nurse knows that the patient is at increased risk for what complication of his injury? A) Hematoma B) Skull fracture C) Embolus D) Stroke

A Feedback: Two major factors place older adults at increased risk for hematomas. First, the dura becomes more adherent to the skull with increasing age. Second, many older adults take aspirin and anticoagulants as part of routine management of chronic conditions. Because of these factors, the patient's risk of a hematoma is likely greater than that of stroke, embolism, or skull fracture.

6. A patient who just suffered a suspected ischemic stroke is brought to the ED by ambulance. On what should the nurse's primary assessment focus? A) Cardiac and respiratory status B) Seizure activity C) Pain D) Fluid and electrolyte balance

A Feedback: Acute care begins with managing ABCs. Patients may have difficulty keeping an open and clear airway secondary to decreased LOC. Neurologic assessment with close monitoring for signs of increased neurologic deficit and seizure activity occurs next. Fluid and electrolyte balance must be controlled carefully with the goal of adequate hydration to promote perfusion and decrease further brain activity.

31. The nurse is planning the care of a patient with a T1 spinal cord injury. The nurse has identified the diagnosis of "risk for impaired skin integrity." How can the nurse best address this risk? A) Change the patient's position frequently. B) Provide a high-protein diet. C) Provide light massage at least daily. D) Teach the patient deep breathing and coughing exercises.

A Feedback: Frequent position changes are among the best preventative measures against pressure ulcers. A high-protein diet can benefit wound healing, but does not necessarily prevent skin breakdown. Light massage and deep breathing do not protect or restore skin integrity.

A patient diagnosed with Bells palsy is being cared for on an outpatient basis. During health education, the nurse should promote which of the following actions? A) Applying a protective eye shield at night B) Chewing on the affected side to prevent unilateral neglect C) Avoiding the use of analgesics whenever possible D) Avoiding brushing the teeth

A) Applying a protective eye shield at night Corneal irritation and ulceration may occur if the eye is unprotected. While paralysis lasts, the involved eye must be protected. The patient should be encouraged to eat on the unaffected side, due to swallowing difficulties. Analgesics are used to control the facial pain. The patient should continue to provide self-care including oral hygiene.

The nurse is caring for a patient who is hospitalized with an exacerbation of MS. To ensure the patients safety, what nursing action should be performed? A) Ensure that suction apparatus is set up at the bedside. B) Pad the patients bed rails. C) Maintain bed rest whenever possible. D) Provide several small meals each day.

A) Ensure that suction apparatus is set up at the bedside. Because of the patients risk of aspiration, it is important to have a suction apparatus at hand. Bed rest should be generally be minimized, not maximized, and there is no need to pad the patients bed rails or to provide multiple small meals.

A patient with MS has been admitted to the hospital following an acute exacerbation. When planning the patients care, the nurse addresses the need to enhance the patients bladder control. What aspect of nursing care is most likely to meet this goal? A) Establish a timed voiding schedule. B) Avoid foods that change the pH of urine. C) Perform intermittent catheterization q6h. D) Administer anticholinergic drugs as ordered.

A) Establish a timed voiding schedule. A timed voiding schedule addresses many of the challenges with urinary continence that face the patient with MS. Interventions should be implemented to prevent the need for catheterization and anticholinergics are not normally used.

The nurse is teaching a patient with Guillain-Barr syndrome about the disease. The patient asks how he can ever recover if demyelination of his nerves is occurring. What would be the nurses best response? A) Guillain-Barr spares the Schwann cell, which allows for remyelination in the recovery phase of the disease. B) In Guillain-Barr, Schwann cells replicate themselves before the disease destroys them, so remyelination is possible. C) I know you understand that nerve cells do not remyelinate, so the physician is the best one to answer your question. D) For some reason, in Guillain-Barr, Schwann cells become activated and take over the remyelination process.

A) Guillain-Barr spares the Schwann cell, which allows for remyelination in the recovery phase of the disease. Myelin is a complex substance that covers nerves, providing insulation and speeding the conduction of impulses from the cell body to the dendrites. The cell that produces myelin in the peripheral nervous system is the Schwann cell. In Guillain-Barr syndrome, the Schwann cell is spared, allowing for remyelination in the recovery phase of the disease. The nurse should avoid downplaying the patients concerns by wholly deferring to the physician.

You are the clinic nurse caring for a patient with a recent diagnosis of myasthenia gravis. The patient has begun treatment with pyridostigmine bromide (Mestinon). What change in status would most clearly suggest a therapeutic benefit of this medication? A) Increased muscle strength B) Decreased pain C) Improved GI function D) Improved cognition

A) Increased muscle strength The goal of treatment using pyridostigmine bromide is improvement of muscle strength and control of fatigue. The drug is not intended to treat pain, or cognitive or GI functions.

A 35-year-old woman is diagnosed with a peripheral neuropathy. When making her plan of care, the nurse knows to include what in patient teaching? Select all that apply. A) Inspect the lower extremities for skin breakdown. B) Footwear needs to be accurately sized. C) Immediate family members should be screened for the disease. D) Assistive devices may be needed to reduce the risk of falls. E) Dietary modifications are likely necessary.

A) Inspect the lower extremities for skin breakdown. B) Footwear needs to be accurately sized. D) Assistive devices may be needed to reduce the risk of falls. The plan of care includes inspection of the lower extremities for skin breakdown. Footwear should be accurately sized. Assistive devices, such as a walker or cane, may decrease the risk of falls. Bath water temperature is checked to avoid thermal injury. Peripheral neuropathies do not have a genetic component and diet is unrelated.

The nurse is working with a patient who is newly diagnosed with MS. What basic information should the nurse provide to the patient? A) MS is a progressive demyelinating disease of the nervous system. B) MS usually occurs more frequently in men. C) MS typically has an acute onset. D) MS is sometimes caused by a bacterial infection.

A) MS is a progressive demyelinating disease of the nervous system. MS is a chronic, degenerative, progressive disease of the central nervous system, characterized by the occurrence of small patches of demyelination in the brain and spinal cord. The cause of MS is not known, and the disease affects twice as many women as men.

The nurse is caring for a 77-year-old woman with MS. She states that she is very concerned about the progress of her disease and what the future holds. The nurse should know that elderly patients with MS are known to be particularly concerned about what variables? Select all that apply. A) Possible nursing home placement B) Pain associated with physical therapy C) Increasing disability D) Becoming a burden on the family E) Loss of appetite

A) Possible nursing home placement C) Increasing disability D) Becoming a burden on the family Elderly patients with MS are particularly concerned about increasing disability, family burden, marital concern, and the possible future need for nursing home care. Older adults with MS are not noted to have particular concerns regarding the pain of therapy or loss of appetite.

A patient with diabetes presents to the clinic and is diagnosed with a mononeuropathy. This patients nursing care should involve which of the following? A) Protection of the affected limb from injury B) Passive and active ROM exercises for the affected limb C) Education about improvements to glycemic control D) Interventions to prevent contractures

A) Protection of the affected limb from injury Nursing care involves protection of the affected limb or area from injury, as well as appropriate patient teaching about mononeuropathy and its treatment. Nursing care for this patient does not likely involve exercises or assistive devices, since these are unrelated to the etiology of the disease. Improvements to diabetes management may or may not be necessary.

The nurse is developing a plan of care for a patient with Guillain-Barr syndrome. Which of the following interventions should the nurse prioritize for this patient? A) Using the incentive spirometer as prescribed B) Maintaining the patient on bed rest C) Providing aids to compensate for loss of vision D) Assessing frequently for loss of cognitive function

A) Using the incentive spirometer as prescribed Respiratory function can be maximized with incentive spirometry and chest physiotherapy. Nursing interventions toward enhancing physical mobility should be utilized. Nursing interventions are aimed at preventing a deep vein thrombosis. Guillain-Barr syndrome does not affect cognitive function or vision.

The nurse is planning discharge education for a patient with trigeminal neuralgia. The nurse knows to include information about factors that precipitate an attack. What would the nurse be correct in teaching the patient to avoid? A) Washing his face B) Exposing his skin to sunlight C) Using artificial tears D) Drinking large amounts of fluids

A) Washing his face Washing the face should be avoided if possible because this activity can trigger an attack of pain in a patient with trigeminal neuralgia. Using artificial tears would be an appropriate behavior. Exposing the skin to sunlight would not be harmful to this patient. Temperature extremes in beverages should be avoid

29. A patient with an inoperable brain tumor has been told that he has a short life expectancy. On what aspects of assessment and care should the home health nurse focus? Select all that apply. A) Pain control B) Management of treatment complications C) Interpretation of diagnostic tests D) Assistance with self-care E) Administration of treatments

A, B, D, E Feedback: Home care needs and interventions focus on four major areas: palliation of symptoms and pain control, assistance in self-care, control of treatment complications, and administration of specific forms of treatment, such as parenteral nutrition. Interpretation of diagnostic tests is normally beyond the purview of the nurse.

24. The nurse caring for a patient in a persistent vegetative state is regularly assessing for potential complications. Complications of neurologic dysfunction for which the nurse should assess include which of the following? Select all that apply. A) Contractures B) Hemorrhage C) Pressure ulcers D) Venous thromboembolism E) Pneumonia

A, C, D, E Feedback: Based on the assessment data, potential complications may include respiratory distress or failure, pneumonia, aspiration, pressure ulcer, deep vein thrombosis (DVT), and contractures. The pathophysiology of decreased LOC does not normally create a heightened risk for hemorrhage.

36. During a patient's recovery from stroke, the nurse should be aware of predictors of stroke outcome in order to help patients and families set realistic goals. What are the predictors of stroke outcome? Select all that apply. A) National Institutes of Health Stroke Scale (NIHSS) score B) Race C) LOC at time of admission D) Gender E) Age

A, C, E Feedback: It is helpful for clinicians to be knowledgeable about the relative importance of predictors of stroke outcome (age, NIHSS score, and LOC at time of admission) to provide stroke survivors and their families with realistic goals. Race and gender are not predictors of stroke outcome.

28. The school nurse is giving a presentation on preventing spinal cord injuries (SCI). What should the nurse identify as prominent risk factors for SCI? Select all that apply. A) Young age B) Frequent travel C) African American race D) Male gender E) Alcohol or drug use

A, D, E Feedback: The predominant risk factors for SCI include young age, male gender, and alcohol and drug use. Ethnicity and travel are not risk factors.

A nurse is reviewing postoperative instructions with a client following cataract surgery. Which of the following client statements indicates an understanding of the instructions? A. "I should call my doctor if my vision gets worse" B. "I will take aspirin for eye discomfort" C. "I can blow my nose to clear out any drainage" D. "I can lift objects up to 20 lbs"

A. "I should call my doctor if my vision gets worse."- The client should expect an improvement in vision after the surgery, so the nurse should instruct the client to report negative changes in vision immediately. "I will take aspirin for eye discomfort."The client should avoid aspirin because it can cause bleeding in the eye. "I can blow my nose to clear out any drainage."The client should avoid blowing their nose because it can increase intraocular pressure. "I can lift objects up to 20 pounds."The client should avoid lifting objects heavier than 4.5 kg (10 lb) because it can increase intraocular pressure.

A nurse is developing a teaching plan for a client who has Meniere's disease. Which of the following instructions should the nurse include? A. "Move your head slowly to decrease vertigo" B. "Apply warm packs to the affected ear during acute attacks" C. "Increase your intake of foods and fluids high in salt" D. "Take corticosteroid during acute attacks"

A. "Move your head slowly to decrease vertigo." The nurse should instruct the client to use slow head movements to keep from worsening the vertigo. "Apply warm packs to the affected ear during acute attacks."-Applying warm packs to the affected ear does not relieve the manifestations of Ménière's disease. Helpful interventions include drinking plenty of water, decreasing salt intake, and not smoking. "Increase your intake of foods and fluids high in salt."Clients who have Ménière's disease should avoid consuming foods and fluids that have a high sodium content because they cause fluid retention, which exacerbates the manifestations of Ménière's disease. "Take corticosteroids during acute attacks."Taking corticosteroids will not relieve the manifestations of Ménière's disease and can actually worsen them because these medications cause fluid retention. The client should take an antihistamine, such as meclizine, to minimize or stop the attack.

Which of the following is contraindicated in a patient with increased ICP?* A. Lumbar puncture B. Midline position of the head C. Hyperosmotic diuretics D. Barbiturates medications

A. Lumbar puncture The answer is A. LPs are avoided in patients with ICP because they can lead to possible brain herniation.

You're collecting vital signs on a patient with ICP. The patient has a Glascoma Scale rating of 4. How will you assess the patient's temperature? A. Rectal B. Oral C. Axillary

A. Rectal The answer is A. This GCS rating demonstrates the patient is unconscious. If a patient is unconscious the nurse should take the patient's temperature either via the rectal, tympanic, or temporal method. Oral and axillary are not reliable.

A nurse is planning care for a client who has a closed traumatic brain injury from a fall and is receiving mechanical ventilation. Which of the following intervention is the nurse's priority? A. Maintain a PaCO2 of approx. 35 mmHg B. Provide small doses of fentanyl via bolus for pain management C. Measure body temperature every 1-2hr

A. Maintain a PaCO2 of approx. 35 mmHg The greatest risk to this client is injury from increased intracranial pressure. Therefore, the nurse's priority action is to maintain the PaCO2 at approximately 35 mm Hg to prevent hypercarbia and subsequent vasodilation effects that lead to increase in intracranial pressure.

A nurse in an emergency department is assessing a client who reports sudden, severe eye pain with blurry vision. The provider determines that the client has primary angle-closure glaucoma. Which of the following medications should the nurse expect to administer? A. Osmotic diuretics via IV bolus B. Mydriatic ophthalmic drops C. Corticosteroid ophthalmic drops D. Epinephrine via IV bolus

A. Osmotic diuretics via IV bolus The nurse should expect to administer prescribed osmotic diuretics, such as mannitol, to reduce intraocular pressure and prevent damage to the eye. Mydriatic ophthalmic dropsClients who have primary angle-closure glaucoma should not receive mydriatic ophthalmic drops because they cause pupillary dilation. Instead, the nurse should expect to administer medications that decrease intraocular pressure by increasing the absorption or decreasing the production of aqueous humor. Corticosteroid ophthalmic drops-Corticosteroid ophthalmic drops are used for inflammatory conditions of the eye, such as conjunctivitis. There is no indication for clients who have primary angle-closure glaucoma to receive corticosteroid ophthalmic drops. Instead, the nurse should expect to administer medications that decrease intraocular pressure by increasing the absorption or decreasing the production of aqueous humor. Epinephrine via IV bolusClients who have primary angle-closure glaucoma should not receive epinephrine-containing medications because they cause vasoconstriction. Instead, the nurse should expect to administer medications that decrease intraocular pressure by increasing the absorption or decreasing the production of aqueous humor.

A nurse is assessing a client who has a head injury following a motor-vehicle crash. The nurse should identify that which of the following findings indicates increasing intracranial pressure? A. Restlessness B. Dizziness C. HPTN D. Fever

A. Restlessness- Behavioral changes, such as restlessness and irritability, are early manifestations of increased intracranial pressure. DizzinessAlthough dizziness might be present after head trauma, it is not a manifestation of increased intracranial pressure. HypotensionAlthough hypotension might be present after head trauma, especially if the client is experiencing hypovolemic or neurogenic shock, it is not a manifestation of increased intracranial pressure. Cushing's triad of hypertension, bradycardia, and a widening pulse pressure is a late manifestation of increased intracranial pressure. FeverAlthough a client who has head trauma can develop fever, it is either in response to infection or due to hypothalamic damage, not due to increased intracranial pressure.

A nurse in an emergency department is caring for a client who suddenly lost consciousness and fell wile at home. The provider determines that the client had an embolic stroke. Which of the following medications should the nurse expect to administer? A. Tissue plasminogen activator B. Recombinant factor VIII C. Nitroglycerin D. Lidocaine

A. Tissue plasminogen activator- Tissue plasminogen activator is a thrombolytic agent that should dissolve the blood clot that caused the stroke. Recombinant factor VIII- Recombinant factor VIII helps manage the manifestations of hemophilia. Nitroglycerin-Nitroglycerin is a coronary and venous vasodilator that treats angina. Lidocaine- Lidocaine is an antidysrhythmic agent that treats ventricular dysrhythmias

The nurse is developing a plan of care for a patient newly diagnosed with Bells palsy. The nurses plan of care should address what characteristic manifestation of this disease? A) Tinnitus B) Facial paralysis C) Pain at the base of the tongue D) Diplopia

B) Facial paralysis Bells palsy is characterized by facial dysfunction, weakness, and paralysis. It does not result in diplopia, pain at the base of the tongue, or tinnitus.

The nurse is evaluating the collaborative care of a client with traumatic brain injury (TBI). What is the most important goal for this client?

Achieving the highest level of functioning The nurse's goal for the client with TBI is to help him or her achieve the highest level of functioning possible. The nurse assesses cerebral perfusion, such as oxygenation status, but cannot increase cerebral perfusion. Prevention of injury from falls, infection, or further impairment of cerebral perfusion is part of a larger goal for this client. Prevention of respiratory distress is also part of a larger goal for this client.

a nurse is providing teaching to a client who has a new diagnosis of myasthenia gravis (MG). which of the following pieces of information should the nurse include

set an alarm to ensure medication dosages are taken on time eat high calorie diet avoid over heating

A client hospitalized for hypertension presses the call light and reports "feeling funny." When the nurse gets to the room, the client is slurring words and has right-sided weakness. What does the nurse do first?

Assesses airway, breathing, and circulation The client must be evaluated within 10 minutes of having a stroke. The priority is assessment of the "ABCs"—airway, breathing, and circulation. Calling the Rapid Response Team, not the provider, after assessing ABCs would be appropriate. A neurologic check may be performed later, but is not the priority in this situation. A sitting position is used for hyperreflexia in the client with spinal cord injury to assist in lowering blood pressure.

The nurse is assessing a client with a traumatic brain injury after a skateboarding accident. Which symptom is the nurse most concerned about?

Asymmetric pupils Asymmetric (uneven) pupils, loss of light reaction, or unilateral or bilateral dilated pupils are a sign of a severe traumatic brain injury. Pupil changes are treated as herniation of the brain from increased intracranial pressure (ICP) until proven differently. The nurse should report and document any changes in pupil size, shape, and reactivity to the health care provider immediately because they could indicate an increase in ICP. Amnesia, a head laceration, and restlessness can be symptoms of mild traumatic brain injuries.

12. The nurse is participating in the care of a patient with increased ICP. What diagnostic test is contraindicated in this patient's treatment? A) Computed tomography (CT) scan B) Lumbar puncture C) Magnetic resonance imaging (MRI) D) Venous Doppler studies

B Feedback: A lumbar puncture in a patient with increased ICP may cause the brain to herniate from the withdrawal of fluid and change in pressure during the lumbar puncture. Herniation of the brain is a dire and frequently fatal event. CT, MRI, and venous Doppler are considered noninvasive procedures and they would not affect the ICP itself.

38. A nurse is caring for a patient who experiences debilitating cluster headaches. The patient should be taught to take appropriate medications at what point in the course of the onset of a new headache? A) As soon as the patient's pain becomes unbearable B) As soon as the patient senses the onset of symptoms C) Twenty to 30 minutes after the onset of symptoms D) When the patient senses his or her symptoms peaking

B Feedback: A migraine or a cluster headache in the early phase requires abortive medication therapy instituted as soon as possible. Delaying medication administration would lead to unnecessary pain.

34. The nurse is caring for a patient diagnosed with Parkinson's disease. The patient is having increasing problems with rising from the sitting to the standing position. What should the nurse suggest to the patient to use that will aid in getting from the sitting to the standing position as well as aid in improving bowel elimination? A) Use of a bedpan B) Use of a raised toilet seat C) Sitting quietly on the toilet every 2 hours D) Following the outlined bowel program

B Feedback: A raised toilet seat is useful, because the patient has difficulty in moving from a standing to a sitting position. A handicapped toilet is not high enough and will not aid in improving bowel elimination. Sitting quietly on the toilet every 2 hours will not aid in getting from the sitting to standing position; neither will following the outlined bowel program.

18. A patient has just returned to the unit from the PACU after surgery for a tumor within the spine. The patient complains of pain. When positioning the patient for comfort and to reduce injury to the surgical site, the nurse will position to patient in what position? A) In the high Fowler's position B) In a flat side-lying position C) In the Trendelenberg position D) In the reverse Trendelenberg position

B Feedback: After spinal surgery, the bed is usually kept flat initially. The side-lying position is usually the most comfortable because this position imposes the least pressure on the surgical site. The Fowler's position, Trendelenberg position, and reverse Trendelenberg position are inappropriate for this patient because they would result in increased pain and complications.

16. A clinic nurse is caring for a patient diagnosed with migraine headaches. During the patient teaching session, the patient questions the nurse regarding alcohol consumption. What would the nurse be correct in telling the patient about the effects of alcohol? A) Alcohol causes hormone fluctuations. B) Alcohol causes vasodilation of the blood vessels. C) Alcohol has an excitatory effect on the CNS. D) Alcohol diminishes endorphins in the brain.

B Feedback: Alcohol causes vasodilation of the blood vessels and may exacerbate migraine headaches. Alcohol has a depressant effect on the CNS. Alcohol does not cause hormone fluctuations, nor does it decrease endorphins (morphine-like substances produced by the body) in the brain.

4. When caring for a patient who had a hemorrhagic stroke, close monitoring of vital signs and neurologic changes is imperative. What is the earliest sign of deterioration in a patient with a hemorrhagic stroke of which the nurse should be aware? A) Generalized pain B) Alteration in level of consciousness (LOC) C) Tonicclonic seizures D) Shortness of breath

B Feedback: Alteration in LOC is the earliest sign of deterioration in a patient after a hemorrhagic stroke, such as mild drowsiness, slight slurring of speech, and sluggish papillary reaction. Sudden headache may occur, but generalized pain is less common. Seizures and shortness of breath are not identified as early signs of hemorrhagic stroke. 5. The nurse is performing stroke risk screenings at a hospital open house. The nurse has identified four patients who might be at risk for a stroke. Which patient is likely at the highest risk for a hemorrhagic stroke? A) White female, age 60, with history of excessive alcohol intake B) White male, age 60, with history of uncontrolled hypertension C) Black male, age 60, with history of diabetes D) Black male, age 50, with history of smoking [B Feedback: Uncontrolled hypertension is the primary cause of a hemorrhagic stroke. Control of hypertension, especially in individuals over 55 years of age, clearly reduces the risk for hemorrhagic stroke. Additional risk factors are increased age, male gender, and excessive alcohol intake. Another high-risk group includes African Americans, where the incidence of first stroke is almost twice that as in Caucasians.

16. An elderly woman found with a head injury on the floor of her home is subsequently admitted to the neurologic ICU. What is the best rationale for the following physician orders: elevate the HOB; keep the head in neutral alignment with no neck flexion or head rotation; avoid sharp hip flexion? A) To decrease cerebral arterial pressure B) To avoid impeding venous outflow C) To prevent flexion contractures D) To prevent aspiration of stomach contents

B Feedback: Any activity or position that impedes venous outflow from the head may contribute to increased volume inside the skull and possibly increase ICP. Cerebral arterial pressure will be affected by the balance between oxygen and carbon dioxide. Flexion contractures are not a priority at this time. Stomach contents could still be aspirated in this position.

3. A nurse is caring for a critically ill patient with autonomic dysreflexia. What clinical manifestations would the nurse expect in this patient? A) Respiratory distress and projectile vomiting B) Bradycardia and hypertension C) Tachycardia and agitation D) Third-spacing and hyperthermia

B Feedback: Autonomic dysreflexia is characterized by a pounding headache, profuse sweating, nasal congestion, piloerection ("goose bumps"), bradycardia, and hypertension. It occurs in cord lesions above T6 after spinal shock has resolved; it does not result in vomiting, tachycardia, or third-spacing.

38. The nurse is caring for a patient who is scheduled for a cervical discectomy the following day. During health education, the patient should be made aware of what potential complications? A) Vertebral fracture B) Hematoma at the surgical site C) Scoliosis D) Renal trauma

B Feedback: Based on all the assessment data, the potential complications of diskectomy may include hematoma at the surgical site, resulting in cord compression and neurologic deficit and recurrent or persistent pain after surgery. Renal trauma and fractures are unlikely; scoliosis is a congenital malformation of the spine.

35. The nurse is caring for a patient recovering from an ischemic stroke. What intervention best addresses a potential complication after an ischemic stroke? A) Providing frequent small meals rather than three larger meals B) Teaching the patient to perform deep breathing and coughing exercises C) Keeping a urinary catheter in situ for the full duration of recovery D) Limiting intake of insoluble fiber Ans:

B Feedback: Because pneumonia is a potential complication of stroke, deep breathing and coughing exercises should be encouraged unless contraindicated. No particular need exists to provide frequent meals and normally fiber intake should not be restricted. Urinary catheters should be discontinued as soon as possible.

31. A preceptor is discussing stroke with a new nurse on the unit. The preceptor would tell the new nurse which cardiac dysrhythmia is associated with cardiogenic embolic strokes? A) Ventricular tachycardia B) Atrial fibrillation C) Supraventricular tachycardia D) Bundle branch block

B Feedback: Cardiogenic embolic strokes are associated with cardiac dysrhythmias, usually atrial fibrillation. The other listed dysrhythmias are less commonly associated with this type of stroke.

12. A patient has been admitted to the ICU after being recently diagnosed with an aneurysm and the patient's admission orders include specific aneurysm precautions. What nursing action will the nurse incorporate into the patient's plan of care? A) Elevate the head of the bed to 45 degrees. B) Maintain the patient on complete bed rest. C) Administer enemas when the patient is constipated. D) Avoid use of thigh-high elastic compression stockings.

B Feedback: Cerebral aneurysm precautions are implemented for the patient with a diagnosis of aneurysm to provide a nonstimulating environment, prevent increases in ICP, and prevent further bleeding. The patient is placed on immediate and absolute bed rest in a quiet, nonstressful environment because activity, pain, and anxiety elevate BP, which increases the risk for bleeding. Visitors, except for family, are restricted. The head of the bed is elevated 15 to 30 degrees to promote venous drainage and decrease ICP. Some neurologists, however, prefer that the patient remains flat to increase cerebral perfusion. No enemas are permitted, but stool softeners and mild laxatives are prescribed. Thigh-high elastic compression stockings or sequential compression boots may be ordered to decrease the patient's risk for deep vein thrombosis (DVT).

23. A patient has recently begun mobilizing during the recovery from an ischemic stroke. To protect the patient's safety during mobilization, the nurse should perform what action? A) Support the patient's full body weight with a waist belt during ambulation. B) Have a colleague follow the patient closely with a wheelchair. C) Avoid mobilizing the patient in the early morning or late evening. D) Ensure that the patient's family members do not participate in mobilization.

B Feedback: During mobilization, a chair or wheelchair should be readily available in case the patient suddenly becomes fatigued or feels dizzy. The family should be encouraged to participate, as appropriate, and the nurse should not have to support the patient's full body weight. Morning and evening activity are not necessarily problematic. 24. A patient diagnosed with a hemorrhagic stroke has been admitted to the neurologic ICU. The nurse knows that teaching for the patient and family needs to begin as soon as the patient is settled on the unit and will continue until the patient is discharged. What will family education need to include? A) How to differentiate between hemorrhagic and ischemic stroke B) Risk factors for ischemic stroke C) How to correctly modify the home environment D) Techniques for adjusting the patient's medication dosages at home [C Feedback: For a patient with a hemorrhagic stroke, teaching addresses the use of assistive devices or modification of the home environment to help the patient live with the disability. This is more important to the patient's needs than knowing about risk factors for ischemic stroke. It is not necessary for the family to differentiate between different types of strokes. Medication regimens should never be altered without consultation.

13. A nurse is caring for a patient diagnosed with a hemorrhagic stroke. When creating this patient's plan of care, what goal should be prioritized? A) Prevent complications of immobility. B) Maintain and improve cerebral tissue perfusion. C) Relieve anxiety and pain. D) Relieve sensory deprivation.

B Feedback: Each of the listed goals is appropriate in the care of a patient recovering from a stroke. However, promoting cerebral perfusion is a priority physiologic need, on which the patient's survival depends.

17. When caring for a patient who has had a stroke, a priority is reduction of ICP. What patient position is most consistent with this goal? A) Head turned slightly to the right side B) Elevation of the head of the bed C) Position changes every 15 minutes while awake D) Extension of the neck

B Feedback: Elevation of the head of the bed promotes venous drainage and lowers ICP; the nurse should avoid flexing or extending the neck or turning the head side to side. The head should be in a neutral midline position. Excessively frequent position changes are unnecessary.

14. A patient diagnosed with a pituitary adenoma has arrived on the neurologic unit. When planning the patient's care, the nurse should be aware that the effects of the tumor will primarily depend on what variable? A) Whether the tumor utilizes aerobic or anaerobic respiration B) The specific hormones secreted by the tumor C) The patient's pre-existing health status D) Whether the tumor is primary or the result of metastasis

B Feedback: Functioning pituitary tumors can produce one or more hormones normally produced by the anterior pituitary and the effects of the tumor depend largely on the identity of these hormones. This variable is more significant than the patient's health status or whether the tumor is primary versus secondary. Anaerobic and aerobic respiration is not relevant.

2. A patient is brought to the trauma center by ambulance after sustaining a high cervical spinal cord injury 1½ hours ago. Endotracheal intubation has been deemed necessary and the nurse is preparing to assist. What nursing diagnosis should the nurse associate with this procedure? A) Risk for impaired skin integrity B) Risk for injury C) Risk for autonomic dysreflexia D) Risk for suffocation

B Feedback: If endotracheal intubation is necessary, extreme care is taken to avoid flexing or extending the patient's neck, which can result in extension of a cervical injury. Intubation does not directly cause autonomic dysreflexia and the threat to skin integrity is a not a primary concern. Intubation does not carry the potential to cause suffocation.

20. Following a spinal cord injury a patient is placed in halo traction. While performing pin site care, the nurse notes that one of the traction pins has become detached. The nurse would be correct in implementing what priority nursing action? A) Complete the pin site care to decrease risk of infection. B) Notify the neurosurgeon of the occurrence. C) Stabilize the head in a lateral position. D) Reattach the pin to prevent further head trauma.

B Feedback: If one of the pins became detached, the head is stabilized in neutral position by one person while another notifies the neurosurgeon. Reattaching the pin as a nursing intervention would not be done due to risk of increased injury. Pin site care would not be a priority in this instance. Prevention of neurologic injury is the priority.

16. The nurse in an extended care facility is planning the daily activities of a patient with postpolio syndrome. The nurse recognizes the patient will best benefit from physical therapy when it is scheduled at what time? A) Immediately after meals B) In the morning C) Before bedtime D) In the early evening

B Feedback: Important activities for patients with postpolio syndrome should be planned for the morning, as fatigue often increases in the afternoon and evening.

39. A rehabilitation nurse caring for a patient who has had a stroke is approached by the patient's family and asked why the patient has to do so much for herself when she is obviously struggling. What would be the nurse's best answer? A) We are trying to help her be as useful as she possibly can. B) The focus on care in a rehabilitation facility is to help the patient to resume as much self-care as possible. C) We aren't here to care for her the way the hospital staff did; we are here to help her get better so she can go home. D) Rehabilitation means helping patients do exactly what they did before their stroke.

B Feedback: In both acute care and rehabilitation facilities, the focus is on teaching the patient to resume as much self-care as possible. The goal of rehabilitation is not to be useful, nor is it to return patients to their prestroke level of functioning, which may be unrealistic.

10. While completing a health history on a patient who has recently experienced a seizure, the nurse would assess for what characteristic associated with the postictal state? A) Epileptic cry B) Confusion C) Urinary incontinence D) Body rigidity

B Feedback: In the postictal state (after the seizure), the patient is often confused and hard to arouse and may sleep for hours. The epileptic cry occurs from the simultaneous contractions of the diaphragm and chest muscles that occur during the seizure. Urinary incontinence and intense rigidity of the entire body are followed by alternating muscle relaxation and contraction (generalized tonic-clonic contraction) during the seizure.

8. A patient with spinal cord injury has a nursing diagnosis of altered mobility and the nurse recognizes the increased the risk of deep vein thrombosis (DVT). Which of the following would be included as an appropriate nursing intervention to prevent a DVT from occurring? A) Placing the patient on a fluid restriction as ordered B) Applying thigh-high elastic stockings C) Administering an antifibrinolyic agent D) Assisting the patient with passive range of motion (PROM) exercises

B Feedback: It is important to promote venous return to the heart and prevent venous stasis in a patient with altered mobility. Applying elastic stockings will aid in the prevention of a DVT. The patient should not be placed on fluid restriction because a dehydrated state will increase the risk of clotting throughout the body. Antifibrinolytic agents cause the blood to clot, which is absolutely contraindicated in this situation. PROM exercises are not an effective protection against the development of DVT.

9. A patient exhibiting an altered level of consciousness (LOC) due to blunt-force trauma to the head is admitted to the ED. The physician determines the patient's injury is causing increased intracranial pressure (ICP). The nurse should gauge the patient's LOC on the results of what diagnostic tool? A) Monro-Kellie hypothesis B) Glasgow Coma Scale C) Cranial nerve function D) Mental status examination

B Feedback: LOC, a sensitive indicator of neurologic function, is assessed based on the criteria in the Glasgow Coma Scale: eye opening, verbal response, and motor response. The Monro-Kellie hypothesis states that because of the limited space for expansion within the skull, an increase in any one of the components (blood, brain tissue, cerebrospinal fluid) causes a change in the volume of the others. Cranial nerve function and the mental status examination would be part of the neurologic examination for this patient, but would not be the priority in evaluating LOC.

2. The nurse is providing care for a patient who is unconscious. What nursing intervention takes highest priority? A) Maintaining accurate records of intake and output B) Maintaining a patent airway C) Inserting a nasogastric (NG) tube as ordered D) Providing appropriate pain control

B Feedback: Maintaining a patent airway always takes top priority, even though each of the other listed actions is necessary and appropriate.

6. The staff educator is precepting a nurse new to the critical care unit when a patient with a T2 spinal cord injury is admitted. The patient is soon exhibiting manifestations of neurogenic shock. In addition to monitoring the patient closely, what would be the nurse's most appropriate action? A) Prepare to transfuse packed red blood cells. B) Prepare for interventions to increase the patient's BP. C) Place the patient in the Trendelenberg position. D) Prepare an ice bath to lower core body temperature.

B Feedback: Manifestations of neurogenic shock include decreased BP and heart rate. Cardiac markers would be expected to rise in cardiogenic shock. Transfusion, repositioning, and ice baths are not indicated interventions.

13. The nurse is caring for a patient who is in status epilepticus. What medication does the nurse know may be given to halt the seizure immediately? A) Intravenous phenobarbital (Luminal) B) Intravenous diazepam (Valium) C) Oral lorazepam (Ativan) D) Oral phenytoin (Dilantin)

B Feedback: Medical management of status epilepticus includes IV diazepam (Valium) and IV lorazepam (Ativan) given slowly in an attempt to halt seizures immediately. Other medications (phenytoin, phenobarbital) are given later to maintain a seizure-free state. Oral medications are not given during status epilepticus.

25. A neurologic flow chart is often used to document the care of a patient with a traumatic brain injury. At what point in the patient's care should the nurse begin to use a neurologic flow chart? A) When the patient's condition begins to deteriorate B) As soon as the initial assessment is made C) At the beginning of each shift D) When there is a clinically significant change in the patient's condition

B Feedback: Neurologic parameters are assessed initially and as frequently as the patient's condition requires. As soon as the initial assessment is made, the use of a neurologic flowchart is started and maintained. A new chart is not begun at the start of every shift.

10. A patient who has sustained a nondepressed skull fracture is admitted to the acute medical unit. Nursing care should include which of the following? A) Preparation for emergency craniotomy B) Watchful waiting and close monitoring C) Administration of inotropic drugs D) Fluid resuscitation

B Feedback: Nondepressed skull fractures generally do not require surgical treatment; however, close observation of the patient is essential. A craniotomy would not likely be needed if the fracture is nondepressed. Even if treatment is warranted, it is unlikely to include inotropes or fluid resuscitation.

2. A 25-year-old female patient with brain metastases is considering her life expectancy after her most recent meeting with her oncologist. Based on the fact that the patient is not receiving treatment for her brain metastases, what is the nurse's most appropriate action? A) Promoting the patient's functional status and ADLs B) Ensuring that the patient receives adequate palliative care C) Ensuring that the family does not tell the patient that her condition is terminal D) Promoting adherence to the prescribed medication regimen

B Feedback: Patients with intracerebral metastases who are not treated have a steady downhill course with a limited survival time, whereas those who are treated may survive for slightly longer periods, but for most cure is not possible. Palliative care is thus necessary. This is a priority over promotion of function and the family should not normally withhold information from the patient. Adherence to medications such as analgesics is important, but palliative care is a high priority.

13. A patient has been admitted to the neurologic unit for the treatment of a newly diagnosed brain tumor. The patient has just exhibited seizure activity for the first time. What is the nurse's priority response to this event? A) Identify the triggers that precipitated the seizure. B) Implement precautions to ensure the patient's safety. C) Teach the patient's family about the relationship between brain tumors and seizure activity. D) Ensure that the patient is housed in a private room.

B Feedback: Patients with seizures are carefully monitored and protected from injury. Patient safety is a priority over health education, even though this is appropriate and necessary. Specific triggers may or may not be evident; identifying these is not the highest priority. A private room is preferable, but not absolutely necessary.

6. The nurse is caring for a boy who has muscular dystrophy. When planning assistance with the patient's ADLs, what goal should the nurse prioritize? A) Promoting the patient's recovery from the disease B) Maximizing the patient's level of function C) Ensuring the patient's adherence to treatment D) Fostering the family's participation in care

B Feedback: Priority for the care of the child with muscular dystrophy is the need to maximize the patient's level of function. Family participation is also important, but should be guided by this goal. Adherence is not a central goal, even though it is highly beneficial, and the disease is not curable.

17. A patient newly diagnosed with a cervical disk herniation is receiving health education from the clinic nurse. What conservative management measures should the nurse teach the patient to implement? A) Perform active ROM exercises three times daily. B) Sleep on a firm mattress. C) Apply cool compresses to the back of the neck daily. D) Wear the cervical collar for at least 2 hours at a time.

B Feedback: Proper positioning on a firm mattress and bed rest for 1 to 2 days may bring dramatic relief from pain. The patient may need to wear a cervical collar 24 hours a day during the acute phase of pain from a cervical disk herniation. Hot, moist compresses applied to the back of the neck will increase blood flow to the muscles and help relax the spastic muscles.

27. A family member brings the patient to the clinic for a follow-up visit after a stroke. The family member asks the nurse what he can do to decrease his chance of having another stroke. What would be the nurse's best answer? A) Have your heart checked regularly. B) Stop smoking as soon as possible. C) Get medication to bring down your sodium levels. D) Eat a nutritious diet.

B Feedback: Smoking is a modifiable and highly significant risk factor for stroke. The significance of smoking, and the potential benefits of quitting, exceed the roles of sodium, diet, and regular medical assessments.

34. A school nurse is called to the playground where a 6-year-old girl has been found unresponsive and "staring into space," according to the playground supervisor. How would the nurse document the girl's activity in her chart at school? A) Generalized seizure B) Absence seizure C) Focal seizure D) Unclassified seizure

B Feedback: Staring episodes characterize an absence seizure, whereas focal seizures, generalized seizures, and unclassified seizures involve uncontrolled motor activity.

18. A nurse is reviewing the trend of a patient's scores on the Glasgow Coma Scale (GCS). This allows the nurse to gauge what aspect of the patient's status? A) Reflex activity B) Level of consciousness C) Cognitive ability D) Sensory involvement

B Feedback: The Glasgow Coma Scale (GCS) examines three responses related to LOC: eye opening, best verbal response, and best motor response.

15. The neurologic ICU nurse is admitting a patient following a craniotomy using the supratentorial approach. How should the nurse best position the patient? A) Position the patient supine. B) Maintain head of bed (HOB) elevated at 30 to 45 degrees. C) Position patient in prone position. D) Maintain bed in Trendelenberg position.

B Feedback: The patient undergoing a craniotomy with a supratentorial (above the tentorium) approach should be placed with the HOB elevated 30 to 45 degrees, with the neck in neutral alignment. Each of the other listed positions would cause a dangerous elevation in ICP.

4. The nurse is caring for a patient who is postoperative following a craniotomy. When writing the plan of care, the nurse identifies a diagnosis of "deficient fluid volume related to fluid restriction and osmotic diuretic use." What would be an appropriate intervention for this diagnosis? A) Change the patient's position as indicated. B) Monitor serum electrolytes. C) Maintain NPO status. D) Monitor arterial blood gas (ABG) values.

B Feedback: The postoperative fluid regimen depends on the type of neurosurgical procedure and is determined on an individual basis. The volume and composition of fluids are adjusted based on daily serum electrolyte values, along with fluid intake and output. Fluids may have to be restricted in patients with cerebral edema. Changing the patient's position, maintaining an NPO status, and monitoring ABG values do not relate to the nursing diagnosis of deficient fluid volume.

5. The clinic nurse caring for a patient with Parkinson's disease notes that the patient has been taking levodopa and carbidopa (Sinemet) for 7 years. For what common side effect of Sinemet would the nurse assesses this patient? A) Pruritus B) Dyskinesia C) Lactose intolerance D) Diarrhea

B Feedback: Within 5 to 10 years of taking levodopa, most patients develop a response to the medication characterized by dyskinesia (abnormal involuntary movements). Another potential complication of long-term dopaminergic medication use is neuroleptic malignant syndrome characterized by severe rigidity, stupor, and hyperthermia. Side effects of long-term Sinemet therapy are not pruritus, lactose intolerance, or diarrhea.

A patient with herpes simplex virus encephalitis (HSV) has been admitted to the ICU. What medication would the nurse expect the physician to order for the treatment of this disease process? A) Cyclosporine (Neoral) B) Acyclovir (Zovirax) C) Cyclobenzaprine (Flexeril) D) Ampicillin (Prinicpen)

B) Acyclovir (Zovirax) Acyclovir (Zovirax) or ganciclovir (Cytovene), antiviral agents, are the medications of choice in the treatment of HSV. The mode of action is the inhibition of viral DNA replication. To prevent relapse, treatment would continue for up to 3 weeks. Cyclosporine is an immunosuppressant and antirheumatic. Cyclobenzaprine is a centrally acting skeletal muscle relaxant. Ampicillin, an antibiotic, is ineffective against viruses.

a nurse is providing teaching to a class about transient ishemic atracks. (TIA). which of the following pieces of information should the nurse include in the teaching

TIA can precede an ischemic stroke

To alleviate pain associated with trigeminal neuralgia, a patient is taking Tegretol (carbamazepine). What health education should the nurse provide to the patient before initiating this treatment? A) Concurrent use of calcium supplements is contraindicated. B) Blood levels of the drug must be monitored. C) The drug is likely to cause hyperactivity and agitation. D) Tegretol can cause tinnitus during the first few days of treatment.

B) Blood levels of the drug must be monitored. Side effects of Tegretol include nausea, dizziness, drowsiness, and aplastic anemia. The patient must also be monitored for bone marrow depression during long-term therapy. Skin discoloration, insomnia, and tinnitus are not side effects of Tegretol.

A patient diagnosed with MS has been admitted to the medical unit for treatment of an MS exacerbation. Included in the admission orders is baclofen (Lioresal). What should the nurse identify as an expected outcome of this treatment? A) Reduction in the appearance of new lesions on the MRI B) Decreased muscle spasms in the lower extremities C) Increased muscle strength in the upper extremities D) Decreased severity and duration of exacerbations

B) Decreased muscle spasms in the lower extremities Baclofen, a g-aminobutyric acid (GABA) agonist, is the medication of choice in treating spasms. It can be administered orally or by intrathecal injection. Avonex and Betaseron reduce the appearance of new lesions on the MRI. Corticosteroids limit the severity and duration of exacerbations. Anticholinesterase agents increase muscle strength in the upper extremities.

A patient diagnosed with myasthenia gravis has been hospitalized to receive plasmapheresis for a myasthenic exacerbation. The nurse knows that the course of treatment for plasmapheresis in a patient with myasthenia gravis is what? A) Every day for 1 week B) Determined by the patients response C) Alternate days for 10 days D) Determined by the patients weight

B) Determined by the patients response The typical course of plasmapheresis consists of daily or alternate-day treatment, and the number of treatments is determined by the patients response.

A 69-year-old patient is brought to the ED by ambulance because a family member found him lying on the floor disoriented and lethargic. The physician suspects bacterial meningitis and admits the patient to the ICU. The nurse knows that risk factors for an unfavorable outcome include what? Select all that apply. A) Blood pressure greater than 140/90 mm Hg B) Heart rate greater than 120 bpm C) Older age D) Low Glasgow Coma Scale E) Lack of previous immunizations

B) Heart rate greater than 120 bpm C) Older age D) Low Glasgow Coma Scale Risks for an unfavorable outcome of meningitis include older age, a heart rate greater than 120 beats/minute, low Glasgow Coma Scale score, cranial nerve palsies, and a positive Gram stain 1 hour after presentation to the hospital. A BP greater than 140/90 mm Hg is indicative of hypertension, but is not necessarily related to poor outcomes related to meningitis. Immunizations are not normally relevant to the course of the disease.

A patient with possible bacterial meningitis is admitted to the ICU. What assessment finding would the nurse expect for a patient with this diagnosis? A) Pain upon ankle dorsiflexion of the foot B) Neck flexion produces flexion of knees and hips C) Inability to stand with eyes closed and arms extended without swaying D) Numbness and tingling in the lower extremities

B) Neck flexion produces flexion of knees and hips Clinical manifestations of bacterial meningitis include a positive Brudzinskis sign. Neck flexion producing flexion of knees and hips correlates with a positive Brudzinskis sign. Positive Homans sign (pain upon dorsiflexion of the foot) and negative Rombergs sign (inability to stand with eyes closed and arms extended) are not expected assessment findings for the patient with bacterial meningitis. Peripheral neuropathy manifests as numbness and tingling in the lower extremities. Again, this would not be an initial assessment to rule out bacterial meningitis.

7. A nurse is admitting a patient with a severe migraine headache and a history of acute coronary syndrome. What migraine medication would the nurse question for this patient? A) Rizatriptan (Maxalt) B) Naratriptan (Amerge) C) Sumatriptan succinate (Imitrex) D) Zolmitriptan (Zomig)

C Feedback: Triptans can cause chest pain and are contraindicated in patients with ischemic heart disease. Maxalt, Amerge, and Zomig are triptans used in routine clinical use for the treatment of migraine headaches.

A patient with MS has developed dysphagia as a result of cranial nerve dysfunction. What nursing action should the nurse consequently perform? A) Arrange for the patient to receive a low residue diet. B) Position the patient upright during feeding. C) Suction the patient following each meal. D) Withhold liquids until the patient has finished eating.

B) Position the patient upright during feeding. Correct, upright positioning is necessary to prevent aspiration in the patient with dysphagia. There is no need for a low-residue diet and suctioning should not be performed unless there is an apparent need. Liquids do not need to be withheld during meals in order to prevent aspiration.

A male patient presents to the clinic complaining of a headache. The nurse notes that the patient is guarding his neck and tells the nurse that he has stiffness in the neck area. The nurse suspects the patient may have meningitis. What is another well-recognized sign of this infection? A) Negative Brudzinskis sign B) Positive Kernigs sign C) Hyperpatellar reflex D) Sluggish pupil reaction

B) Positive Kernigs sign Meningeal irritation results in a number of well-recognized signs commonly seen in meningitis, such as a positive Kernigs sign, a positive Brudzinskis sign, and photophobia. Hyperpatellar reflex and a sluggish pupil reaction are not commonly recognized signs of meningitis.

The nurse is caring for a patient with multiple sclerosis (MS). The patient tells the nurse the hardest thing to deal with is the fatigue. When teaching the patient how to reduce fatigue, what action should the nurse suggest? A) Taking a hot bath at least once daily B) Resting in an air-conditioned room whenever possible C) Increasing the dose of muscle relaxants D) Avoiding naps during the day

B) Resting in an air-conditioned room whenever possible Fatigue is a common symptom of patients with MS. Lowering the body temperature by resting in an air- conditioned room may relieve fatigue; however, extreme cold should be avoided. A hot bath or shower can increase body temperature, producing fatigue. Muscle relaxants, prescribed to reduce spasticity, can cause drowsiness and fatigue. Planning for frequent rest periods and naps can relieve fatigue. Other measures to reduce fatigue in the patient with MS include treating depression, using occupational therapy to learn energy conservation techniques, and reducing spasticity.

A patient with metastatic cancer has developed trigeminal neuralgia and is taking carbamazepine (Tegretol) for pain relief. What principle applies to the administration of this medication? A) Tegretol is not known to have serious adverse effects. B) The patient should be monitored for bone marrow depression. C) Side effects of the medication include renal dysfunction. D) The medication should be first taken in the maximum dosage form to be effective.

B) The patient should be monitored for bone marrow depression. The anticonvulsant agents carbamazepine (Tegretol) and phenytoin (Dilantin) relieve pain in most patients diagnosed with trigeminal neuralgia by reducing the transmission of impulses at certain nerve terminals. Side effects include nausea, dizziness, drowsiness, and aplastic anemia. Carbamazepine should be gradually increased until pain relief is obtained.

19. The nurse is caring for a patient who is rapidly progressing toward brain death. The nurse should be aware of what cardinal signs of brain death? Select all that apply. A) Absence of pain response B) Apnea C) Coma D) Absence of brain stem reflexes E) Absence of deep tendon reflexes

B, C, D Feedback: The three cardinal signs of brain death upon clinical examination are coma, the absence of brain stem reflexes, and apnea. Absences of pain response and deep tendon reflexes are not necessarily indicative of brain death.

33. As a member of the stroke team, the nurse knows that thrombolytic therapy carries the potential for benefit and for harm. The nurse should be cognizant of what contraindications for thrombolytic therapy? Select all that apply. A) INR above 1.0 B) Recent intracranial pathology C) Sudden symptom onset D) Current anticoagulation therapy E) Symptom onset greater than 3 hours prior to admission

B, D, E Feedback: Some of the absolute contraindications for thrombolytic therapy include symptom onset greater than 3 hours before admission, a patient who is anticoagulated (with an INR above 1.7), or a patient who has recently had any type of intracranial pathology (e.g., previous stroke, head injury, trauma).

A nurse is teaching a client who has multiple sclerosis and has a new prescription for glatiramer acetate. Which of the following statements indicates that the client understands the teaching? A. "I will ask my partner to give the injection in the same spot each time" B. "I will avoid going to the store when it is crowded" C. "I will see relief of my symptoms in about 1 wk" D. " I will exercise rigorously while taking this medication"

B. "I will avoid going to the store when it is crowded."- Clients who are prescribed this medication are instructed to avoid crowds and individuals who have infection. "I will ask my partner to give the injection in the same spot each time." Clients are instructed to rotate the site of injection because local skin reactions are common. "I will see relief of my symptoms in about 1 week."Clients are instructed that it may take up to 6 months for the immune response to become evident. "I will exercise rigorously while taking this medication."Clients who have multiple sclerosis are instructed to avoid activities that increase their temperature, which leads to fatigue.

A nurse is teaching a client who has osteoporosis and has a new prescription for alendronate. Which of the following information should the nurse include in the teaching? A. "Take this medication with 8 oz of milk" B. "Remain upright for 30 min after taking this medication" C. "Wait 1 hr after taking other medications to take alendronate" D. "Take vitamin C to promote absorption of this medicaton"

B. "Remain upright for 30 minutes after taking this medication."- To prevent esophagitis or esophageal ulcers, which can result from alendronate therapy, the client should sit upright for 30 min after taking this medication. "Take this medication with 8 ounces of milk." The nurse should instruct the client to take alendronate with 240 mL (8 oz) of water, not milk. Foods or beverages containing calcium can reduce medication absorption. "Wait 1 hour after taking other medications to take alendronate."The nurse should instruct the client to take alendronate first thing in the morning, at least 30 min before other medications. "Take vitamin C to promote absorption of this medication."Vitamin C intake does not increase alendronate absorption and some sources, such as orange juice, decrease absorption. However, the nurse should encourage the client to take vitamin D, which promotes calcium absorption.

You're providing education to a group of nursing students about ICP. You explain that when cerebral perfusion pressure falls too low the brain is not properly perfused and brain tissue dies. A student asks, "What is a normal cerebral perfusion pressure level?" Your response is:* A. 5-15 mmHg B. 60-100 mmHg C. 30-45 mmHg D. >160 mmHg

B. 60-100 mmHg The answer is B. This is a normal CPP. Option A represents a normal intracranial pressure.

Which patient below is at MOST risk for increased intracranial pressure?* A. A patient who is experiencing severe hypotension. B. A patient who is admitted with a traumatic brain injury. C. A patient who recently experienced a myocardial infarction. D. A patient post-op from eye surgery

B. A patient who is admitted with a traumatic brain injury. The answer is B. Remember head trauma, cerebral hemorrhage, hematoma, hydrocephalus, tumor, encephalitis etc. can all increase ICP.

A nurse is caring for a client who is in balanced suspension skeletal traction and reports intermittent muscle spasms. Which of the following actions should the nurse take first? A. Reposition the client B. Check the position of the weights and ropes C. Administer a muscle relaxant D. Provide distraction

B. Check the position of the weights and ropes. The first action the nurse should take when using the nursing process is to assess the client. The nurse should first check the position of the weights and ropes to investigate the cause of the muscle spasms. The weights might be too heavy, or the nurse might need to realign the client Reposition the client.- The nurse should reposition the client to realign him and try to relieve his muscle spasms. However, there is another action the nurse should take first. Administer a muscle relaxant.- The nurse should administer a muscle relaxant to minimize the client's muscle spasms. However, there is another action the nurse should take first. Provide distraction.- The nurse should provide sensory stimulation to help the client keep his focus away from the pain of the spasms. However, there is another action the nurse should take first.

What assessment finding requires immediate intervention if found while a patient is receiving Mannitol?* A. An ICP of 10 mmHg B. Crackles throughout lung fields C. BP 110/72 D. Patient complains of dry mouth and thirst

B. Crackles throughout lung fields The answer is B. Mannitol can cause fluid volume overload that leads to heart failure and pulmonary edema. Crackles in the lung fields represent pulmonary edema and requires immediate intervention. Option A is a normal ICP reading and shows the mannitol is being effective. BP is within normal limits, and dry mouth/thirst will occur with this medication because remember we are trying to dehydrate the brain to keep edema and intracranial pressure decreased.

During the assessment of a patient with increased ICP, you note that the patient's arms are extended straight out and toes pointed downward. You will document this as:* A. Decorticate posturing B. Decerebrate posturing C. Flaccid posturing

B. Decerebrate posturing The answer is B.

External ventricular drains monitor ICP and are inserted where?A. Subarachnoid space B. Lateral Ventricle C. Epidural space D. Right Ventricle

B. Lateral Ventricle The answer is B. External ventricular drains (also called ventriculostomy) are inserted in the lateral ventricle.

A patient is receiving Mannitol for increased ICP. Which statement is INCORRECT about this medication?*A. Mannitol will remove water from the brain and place it in the blood to be removed from the body.B. Mannitol will cause water and electrolyte reabsorption in the renal tubules.C. When a patient receives Mannitol the nurse must monitor the patient for both fluid volume overload and depletion.D. Mannitol is not for patients who are experiencing anuria.

B. Mannitol will cause water and electrolyte reabsorption in the renal tubules. The answer is B. All the other options are correct. Mannitol will PREVENT (not cause) water and electrolytes (specifically sodium and chloride) from being reabsorbed....hence it will leave the body as urine.

A nurse is caring for a client who has a spastic bladder following a spinal cord injury. Which of the following actions should the nurse take to help stimulate micturition? A. Encourage the client to use the Valvalsa maneuver B. Stroke the client's inner thigh C. Perform the Crede maneuver D. Administer a diuretic

B. Stroke the client's inner thigh The nurse should stimulate micturition by stroking the client's inner thigh. Other techniques include pinching the skin above the groin and providing digital anal stimulation. Perform the Credé maneuver.The nurse should apply direct pressure over the client's bladder, also known as the Credé maneuver, to express urine from a flaccid bladder. It is not effective for clients who have a spastic bladder due to the spasticity of the external sphincter. Administer a diuretic.Antispasmodics such as oxybutynin, rather than diuretics, can be effective for treating mild spastic bladder problems. Encourage the client to use the Valsalva maneuver.-The nurse should encourage the client to hold their breath and bear down, also known as the Valsalva maneuver, to express urine from a flaccid bladder. It is not effective for clients who have a spastic bladder due to the spasticity of the external sphincter.

A nurse is assessing a client who has rheumatoid arthritis. Which of the following findings should the nurse expect? A. Unilateral joint involvement B. Ulnar deviation C. Fracture of the spine D. Decreased sedimentation rate

B. Ulnar deviation- A client who has rheumatoid arthritis can experience inflammation in the hand joints that can make them susceptible to deformity from daily use. Ulnar deviation, or lateral deviation of the fingers, can occur from opening jars and other similar motions. Unilateral joint involvement Rheumatoid arthritis usually occurs bilaterally and symmetrically. Osteoarthritis usually occurs unilaterally. Fractures of the spineCompression fractures of the spine are more common in clients who have osteoporosis. Decreased sedimentation rateA client who has rheumatoid arthritis will have an increased sedimentation rate due to the body's response to the inflammatory connective tissue disorder.

The Monro-Kellie hypothesis explains the compensatory relationship among the structures in the skull that play a role with intracranial pressure. Which of the following are NOT compensatory mechanisms performed by the body to decrease intracranial pressure naturally? Select all that apply:* A. Shifting cerebrospinal fluid to other areas of the brain and spinal cord B. Vasodilation of cerebral vessels C. Decreasing cerebrospinal fluid production D. Leaking proteins into the brain barrier

B. Vasodilation of cerebral vessels D. Leaking proteins into the brain barrier The answers are B and D. These are NOT compensatory mechanisms, but actions that will actually increase intracranial pressure. Vasoconstriction (not dilation) decreases blood flow and helps lower ICP. Leaking of protein actually leads to more swelling of the brain tissue. Remember water is attracted to protein (oncotic pressure).

40. A patient is admitted to the neurologic ICU with a C4 spinal cord injury. When writing the plan of care for this patient, which of the following nursing diagnoses would the nurse prioritize in the immediate care of this patient? A) Risk for impaired skin integrity related to immobility and sensory loss B) Impaired physical mobility related to loss of motor function C) Ineffective breathing patterns related to weakness of the intercostal muscles D) Urinary retention related to inability to void spontaneously

C Feedback: A nursing diagnosis related to breathing pattern would be the priority for this patient. A C4 spinal cord injury will require ventilatory support, due to the diaphragm and intercostals being affected. The other nursing diagnoses would be used in the care plan, but not designated as a higher priority than ineffective breathing patterns.

1. A patient has had an ischemic stroke and has been admitted to the medical unit. What action should the nurse perform to best prevent joint deformities? A) Place the patient in the prone position for 30 minutes/day. B) Assist the patient in acutely flexing the thigh to promote movement. C) Place a pillow in the axilla when there is limited external rotation. D) Place patient's hand in pronation.

C Feedback: A pillow in the axilla prevents adduction of the affected shoulder and keeps the arm away from the chest. The prone position with a pillow under the pelvis, not flat, promotes hyperextension of the hip joints, essential for normal gait. To promote venous return and prevent edema, the upper thigh should not be flexed acutely. The hand is placed in slight supination, not pronation, which is its most functional position.

8. A patient with Parkinson's disease is undergoing a swallowing assessment because she has recently developed adventitious lung sounds. The patient's nutritional needs should be met by what method? Total parenteral nutrition (TPN) B) Provision of a low-residue diet C) Semisolid food with thick liquids D) Minced foods and a fluid restriction

C Feedback: A semisolid diet with thick liquids is easier for a patient with swallowing difficulties to consume than is a solid diet. Low-residue foods and fluid restriction are unnecessary and counterproductive to the patient's nutritional status. The patient's status does not warrant TPN.

26. A community health nurse is giving an educational presentation about stroke and heart disease at the local senior citizens center. What nonmodifiable risk factor for stroke should the nurse cite? A) Female gender B) Asian American race C) Advanced age D) Smoking

C Feedback: Advanced age, male gender, and race are well-known nonmodifiable risk factors for stroke. High-risk groups include people older than 55 years of age; the incidence of stroke more than doubles in each successive decade. Men have a higher rate of stroke than that of women. Another high-risk group is African Americans; the incidence of first stroke in African Americans is almost twice that as in Caucasian Americans; Asian American race is not a risk factor. Smoking is a modifiable risk.

1. The ED nurse is caring for a patient who has been brought in by ambulance after sustaining a fall at home. What physical assessment finding is suggestive of a basilar skull fracture? A) Epistaxis B) Periorbital edema C) Bruising over the mastoid D) Unilateral facial numbness

C Feedback: An area of ecchymosis (bruising) may be seen over the mastoid (Battle's sign) in a basilar skull fracture. Numbness, edema, and epistaxis are not directly associated with a basilar skull fracture.

5. A patient is brought to the ED by her family after falling off the roof. A family member tells the nurse that when the patient fell she was "knocked out," but came to and "seemed okay." Now she is complaining of a severe headache and not feeling well. The care team suspects an epidural hematoma, prompting the nurse to prepare for which priority intervention? A) Insertion of an intracranial monitoring device B) Treatment with antihypertensives C) Emergency craniotomy D) Administration of anticoagulant therapy

C Feedback: An epidural hematoma is considered an extreme emergency. Marked neurologic deficit or respiratory arrest can occur within minutes. Treatment consists of making an opening through the skull to decrease ICP emergently, remove the clot, and control the bleeding. Antihypertensive medications would not be a priority. Anticoagulant therapy should not be ordered for a patient who has a cranial bleed. This could further increase bleeding activity. Insertion of an intracranial monitoring device may be done during the surgery, but is not priority for this patient.

40. A patient with a new diagnosis of ischemic stroke is deemed to be a candidate for treatment with tissue plasminogen activator (t-PA) and has been admitted to the ICU. In addition to closely monitoring the patient's cardiac and neurologic status, the nurse monitors the patient for signs of what complication? A) Acute pain B) Septicemia C) Bleeding D) Seizures

C Feedback: Bleeding is the most common side effect of t-PA administration, and the patient is closely monitored for any bleeding. Septicemia, pain, and seizures are much less likely to result from thrombolytic therapy.

28. A patient with a brain tumor has begun to exhibit signs of cachexia. What subsequent assessment should the nurse prioritize? A) Assessment of peripheral nervous function B) Assessment of cranial nerve function C) Assessment of nutritional status D) Assessment of respiratory status

C Feedback: Cachexia is a wasting syndrome of weight loss, muscle atrophy, fatigue, weakness, and significant loss of appetite. Consequently, nutritional assessment is paramount.

20. Following a traumatic brain injury, a patient has been in a coma for several days. Which of the following statements is true of this patient's current LOC? A) The patient occasionally makes incomprehensible sounds. B) The patient's current LOC will likely become a permanent state. C) The patient may occasionally make nonpurposeful movements. D) The patient is incapable of spontaneous respirations.

C Feedback: Coma is a clinical state of unarousable unresponsiveness in which no purposeful responses to internal or external stimuli occur, although nonpurposeful responses to painful stimuli and brain stem reflexes may be present. Verbal sounds, however, are atypical. Ventilator support may or may not be necessary. Comas are not permanent states.

11. A patient with increased ICP has a ventriculostomy for monitoring ICP. The nurse's most recent assessment reveals that the patient is now exhibiting nuchal rigidity and photophobia. The nurse would be correct in suspecting the presence of what complication? A) Encephalitis B) CSF leak C) Meningitis D) Catheter occlusion

C Feedback: Complications of a ventriculostomy include ventricular infectious meningitis and problems with the monitoring system. Nuchal rigidity and photophobia are clinical manifestations of meningitis, but are not suggestive of encephalitis, a CSF leak, or an occluded catheter.

3. The nurse is discharging home a patient who suffered a stroke. He has a flaccid right arm and leg and is experiencing problems with urinary incontinence. The nurse makes a referral to a home health nurse because of an awareness of what common patient response to a change in body image? A) Denial B) Fear C) Depression D) Disassociation

C Feedback: Depression is a common and serious problem in the patient who has had a stroke. It can result from a profound disruption in his or her life and changes in total function, leaving the patient with a loss of independence. The nurse needs to encourage the patient to verbalize feelings to assess the effect of the stroke on self-esteem. Denial, fear, and disassociation are not the most common patient response to a change in body image, although each can occur in some patients.

28. A patient has experienced a seizure in which she became rigid and then experienced alternating muscle relaxation and contraction. What type of seizure does the nurse recognize? A) Unclassified seizure B) Absence seizure C) Generalized seizure D) Focal seizure

C Feedback: Generalized seizures often involve both hemispheres of the brain, causing both sides of the body to react. Intense rigidity of the entire body may occur, followed by alternating muscle relaxation and contraction (generalized tonic-clonic contraction). This pattern of rigidity does not occur in patients who experience unclassified, absence, or focal seizures.

6. A patient who has been on long-term phenytoin (Dilantin) therapy is admitted to the unit. In light of the adverse of effects of this medication, the nurse should prioritize which of the following in the patient's plan of care? A) Monitoring of pulse oximetry B) Administration of a low-protein diet C) Administration of thorough oral hygiene D) Fluid restriction as ordered

C Feedback: Gingival hyperplasia (swollen and tender gums) can be associated with long-term phenytoin (Dilantin) use. Thorough oral hygiene should be provided consistently and encouraged after discharge. Fluid and protein restriction are contraindicated and there is no particular need for constant oxygen saturation monitoring.

31. A patient, brought to the clinic by his wife and son, is diagnosed with Huntington disease. When providing anticipatory guidance, the nurse should address the future possibility of what effect of Huntington disease? A) Metastasis B) Risk for stroke C) Emotional and personality changes D) Pathologic bone fractures

C Feedback: Huntington disease causes profound changes to personality and behavior. It is a nonmalignant disease and stroke is not a central risk. The disease is not associated with pathologic bone fractures.

25. The nurse is caring for a patient with a brain tumor. What drug would the nurse expect to be ordered to reduce the edema surrounding the tumor? A) Solumedrol B) Dextromethorphan C) Dexamethasone D) Furosemide

C Feedback: If a brain tumor is the cause of the increased ICP, corticosteroids (e.g., dexamethasone) help reduce the edema surrounding the tumor. Solumedrol, a steroid, and furosemide, a loop diuretic, are not the drugs of choice in this instance. Dextromethorphan is used in cough medicines.

11. The nurse is caring for a patient diagnosed with an ischemic stroke and knows that effective positioning of the patient is important. Which of the following should be integrated into the patient's plan of care? A) The patient's hip joint should be maintained in a flexed position. B) The patient should be in a supine position unless ambulating. C) The patient should be placed in a prone position for 15 to 30 minutes several times a day. D) The patient should be placed in a Trendelenberg position two to three times daily to promote cerebral perfusion.

C Feedback: If possible, the patient is placed in a prone position for 15 to 30 minutes several times a day. A small pillow or a support is placed under the pelvis, extending from the level of the umbilicus to the upper third of the thigh. This helps to promote hyperextension of the hip joints, which is essential for normal gait, and helps prevent knee and hip flexion contractures. The hip joints should not be maintained in flexion and the Trendelenberg position is not indicated.

39. The nurse responds to the call light of a patient who has had a cervical diskectomy earlier in the day. The patient states that she is having severe pain that had a sudden onset. What is the nurse's most appropriate action? A) Palpate the surgical site. B) Remove the dressing to assess the surgical site. C) Call the surgeon to report the patient's pain. D) Administer a dose of an NSAID.

C Feedback: If the patient experiences a sudden increase in pain, extrusion of the graft may have occurred, requiring reoperation. A sudden increase in pain should be promptly reported to the surgeon. Administration of an NSAID would be an insufficient response and the dressing should not be removed without an order. Palpation could cause further damage.

9. Paramedics have brought an intubated patient to the RD following a head injury due to acceleration-deceleration motor vehicle accident. Increased ICP is suspected. Appropriate nursing interventions would include which of the following? A) Keep the head of the bed (HOB) flat at all times. B) Teach the patient to perform the Valsalva maneuver. C) Administer benzodiazepines on a PRN basis. D) Perform endotracheal suctioning every hour.

C Feedback: If the patient with a brain injury is very agitated, benzodiazepines are the most commonly used sedatives and do not affect cerebral blood flow or ICP. The HOB should be elevated 30 degrees. Suctioning should be done a limited basis, due to increasing the pressure in the cranium. The Valsalva maneuver is to be avoided. This also causes increased ICP.

20. A patient with amyotrophic lateral sclerosis (ALS) is being visited by the home health nurse who is creating a care plan. What nursing diagnosis is most likely for a patient with this condition? A) Chronic confusion B) Impaired urinary elimination C) Impaired verbal communication D) Bowel incontinence

C Feedback: Impaired communication is an appropriate nursing diagnosis; the voice in patients with ALS assumes a nasal sound and articulation becomes so disrupted that speech is unintelligible. Intellectual function is marginally impaired in patients with late ALS. Usually, the anal and bladder sphincters are intact because the spinal nerves that control muscles of the rectum and urinary bladder are not affected.

15. A patient is admitted to the neurologic ICU with a spinal cord injury. When assessing the patient the nurse notes there is a sudden depression of reflex activity in the spinal cord below the level of injury. What should the nurse suspect? A) Epidural hemorrhage B) Hypertensive emergency C) Spinal shock D) Hypovolemia

C Feedback: In spinal shock, the reflexes are absent, BP and heart rate fall, and respiratory failure can occur. Hypovolemia, hemorrhage, and hypertension do not cause this sudden change in neurologic function.

35. A patient with Parkinson's disease is experiencing episodes of constipation that are becoming increasingly frequent and severe. The patient states that he has been achieving relief for the past few weeks by using OTC laxatives. How should the nurse respond? A) "It's important to drink plenty of fluids while you're taking laxatives." B) "Make sure that you supplement your laxatives with a nutritious diet." C) "Let's explore other options, because laxatives can have side effects and create dependency." D) "You should ideally be using herbal remedies rather than medications to promote bowel function."

C Feedback: Laxatives should be avoided in patients with Parkinson's disease due to the risk of adverse effects and dependence. Herbal bowel remedies are not necessarily less risky.

39. The nurse recognizes that a patient with a SCI is at risk for muscle spasticity. How can the nurse best prevent this complication of an SCI? A) Position the patient in a high Fowler's position when in bed. B) Support the knees with a pillow when the patient is in bed. C) Perform passive ROM exercises as ordered. D) Administer NSAIDs as ordered.

C Feedback: Passive ROM exercises can prevent muscle spasticity following SCI. NSAIDs are not used for this purpose. Pillows and sitting upright do not directly address the patient's risk of muscle spasticity.

12. A patient is admitted to the neurologic ICU with a spinal cord injury. In writing the patient's care plan, the nurse specifies that contractures can best be prevented by what action? A) Repositioning the patient every 2 hours B) Initiating range-of-motion exercises (ROM) as soon as the patient initiates C) Initiating (ROM) exercises as soon as possible after the injury D) Performing ROM exercises once a day

C Feedback: Passive ROM exercises should be implemented as soon as possible after injury. It would be inappropriate to wait for the patient to first initiate exercises. Toes, metatarsals, ankles, knees, and hips should be put through a full ROM at least four, and ideally five, times daily. Repositioning alone will not prevent contractures.

3. The nurse is writing a care plan for a patient with brain metastases. The nurse decides that an appropriate nursing diagnosis is "anxiety related to lack of control over the health circumstances." In establishing this plan of care for the patient, the nurse should include what intervention? A) The patient will receive antianxiety medications every 4 hours. B) The patient's family will be instructed on planning the patient's care. C) The patient will be encouraged to verbalize concerns related to the disease and its treatment. D) The patient will begin intensive therapy with the goal of distraction.

C Feedback: Patients need the opportunity to exercise some control over their situation. A sense of mastery can be gained as they learn to understand the disease and its treatment and how to deal with their feelings. Distraction and administering medications will not allow the patient to gain control over anxiety. Delegating planning to the family will not help the patient gain a sense of control and autonomy.

22. The nurse is caring for a patient newly diagnosed with a primary brain tumor. The patient asks the nurse where his tumor came from. What would be the nurse's best response? A) "Your tumor originated from somewhere outside the CNS." B) "Your tumor likely started out in one of your glands." C) "Your tumor originated from cells within your brain itself." D) "Your tumor is from nerve tissue somewhere in your body."

C Feedback: Primary brain tumors originate from cells and structures within the brain. Secondary brain tumors are metastatic tumors that originate somewhere else in the body. The scenario does not indicate that the patient's tumor is a pituitary tumor or a neuroma.

28. The nurse is reviewing the medication administration record of a female patient who possesses numerous risk factors for stroke. Which of the woman's medications carries the greatest potential for reducing her risk of stroke? A) Naproxen 250 PO b.i.d. B) Calcium carbonate 1,000 mg PO b.i.d. C) Aspirin 81 mg PO o.d. D) Lorazepam 1 mg SL b.i.d. PRN

C Feedback: Research findings suggest that low-dose aspirin may lower the risk of stroke in women who are at risk. Naproxen, lorazepam, and calcium supplements do not have this effect. 29. A nurse in the ICU is providing care for a patient who has been admitted with a hemorrhagic stroke. The nurse is performing frequent neurologic assessments and observes that the patient is becoming progressively more drowsy over the course of the day. What is the nurse's best response to this assessment finding? A) Report this finding to the physician as an indication of decreased metabolism. B) Provide more stimulation to the patient and monitor the patient closely. C) Recognize this as the expected clinical course of a hemorrhagic stroke. D) Report this to the physician as a possible sign of clinical deterioration. [D Feedback: Alteration in LOC often is the earliest sign of deterioration in a patient with a hemorrhagic stroke. Drowsiness and slight slurring of speech may be early signs that the LOC is deteriorating. This finding is unlikely to be the result of metabolic changes and it is not expected. Stimulating a patient with an acute stroke is usually contraindicated.

3. The nurse is caring for a patient in the ICU who has a brain stem herniation and who is exhibiting an altered level of consciousness. Monitoring reveals that the patient's mean arterial pressure (MAP) is 60 mm Hg with an intracranial pressure (ICP) reading of 5 mm Hg. What is the nurse's most appropriate action? A) Position the patient in the high Fowler's position as tolerated. B) Administer osmotic diuretics as ordered. C) Participate in interventions to increase cerebral perfusion pressure. D) Prepare the patient for craniotomy.

C Feedback: The cerebral perfusion pressure (CPP) is 55 mm Hg, which is considered low. The normal CPP is 70 to 100 mm Hg. Patients with a CPP of less than 50 mm Hg experience irreversible neurologic damage. As a result, interventions are necessary. A craniotomy is not directly indicated. Diuretics and increased height of bed would exacerbate the patient's condition.

36. Splints have been ordered for a patient who is at risk of developing footdrop following a spinal cord injury. The nurse caring for this patient knows that the splints are removed and reapplied when? A) At the patient's request B) Each morning and evening C) Every 2 hours D) One hour prior to mobility exercises

C Feedback: The feet are prone to footdrop; therefore, various types of splints are used to prevent footdrop. When used, the splints are removed and reapplied every 2 hours.

32. The pathophysiology of an ischemic stroke involves the ischemic cascade, which includes the following steps: 1. Change in pH 2. Blood flow decreases 3. A switch to anaerobic respiration 4. Membrane pumps fail 5. Cells cease to function 6. Lactic acid is generated Put these steps in order in which they occur. A) 635241 B) 352416 C) 236145 D) 162534

C Feedback: The ischemic cascade begins when cerebral blood flow decreases to less than 25 mL per 100 g of blood per minute. At this point, neurons are no longer able to maintain aerobic respiration. The mitochondria must then switch to anaerobic respiration, which generates large amounts of lactic acid, causing a change in the pH. This switch to the less efficient anaerobic respiration also renders the neuron incapable of producing sufficient quantities of adenosine triphosphate (ATP) to fuel the depolarization processes. The membrane pumps that maintain electrolyte balances begin to fail, and the cells cease to function.

A) To decrease cerebral edema B) To prevent seizure activity that is common following a TIA C) To remove atherosclerotic plaques blocking cerebral flow D) To determine the cause of the TIA

C Feedback: The main surgical procedure for select patients with TIAs is carotid endarterectomy, the removal of an atherosclerotic plaque or thrombus from the carotid artery to prevent stroke in patients with occlusive disease of the extracranial arteries. An endarterectomy does not decrease cerebral edema, prevent seizure activity, or determine the cause of a TIA.

1. A patient is being admitted to the neurologic ICU following an acute head injury that has resulted in cerebral edema. When planning this patient's care, the nurse would expect to administer what priority medication? A) Hydrochlorothiazide (HydroDIURIL) B) Furosemide (Lasix) C) Mannitol (Osmitrol) D) Spirolactone (Aldactone)

C Feedback: The osmotic diuretic mannitol is given to dehydrate the brain tissue and reduce cerebral edema. This drug acts by reducing the volume of brain and extracellular fluid. Spirolactone, furosemide, and hydrochlorothiazide are diuretics that are not typically used in the treatment of increased ICP resulting from cerebral edema.

14. The nurse is preparing health education for a patient who is being discharged after hospitalization for a hemorrhagic stroke. What content should the nurse include in this education? A) Mild, intermittent seizures can be expected. B) Take ibuprofen for complaints of a serious headache. C) Take antihypertensive medication as ordered. D) Drowsiness is normal for the first week after discharge.

C Feedback: The patient and family are provided with information that will enable them to cooperate with the care and restrictions required during the acute phase of hemorrhagic stroke and to prepare the patient to return home. Patient and family teaching includes information about the causes of hemorrhagic stroke and its possible consequences. Symptoms of hydrocephalus include gradual onset of drowsiness and behavioral changes. Hypertension is the most serious risk factor, suggesting that appropriate antihypertensive treatment is essential for a patient being discharged. Seizure activity is not normal; complaints of a serious headache should be reported to the physician before any medication is taken. Drowsiness is not normal or expected.

32. A patient is recovering from intracranial surgery that was performed using the transsphenoidal approach. The nurse should be aware that the patient may have required surgery on what neurologic structure? A) Cerebellum B) Hypothalamus C) Pituitary gland D) Pineal gland

C Feedback: The transsphenoidal approach (through the mouth and nasal sinuses) is often used to gain access to the pituitary gland. This surgical approach would not allow for access to the pineal gland, cerebellum, or hypothalamus.

35. A neurologic nurse is reviewing seizures with a group of staff nurses. How should this nurse best describe the cause of a seizure? A) Sudden electrolyte changes throughout the brain B) A dysrhythmia in the peripheral nervous system C) A dysrhythmia in the nerve cells in one section of the brain D) Sudden disruptions in the blood flow throughout the brain

C Feedback: The underlying cause of a seizure is an electrical disturbance (dysrhythmia) in the nerve cells in one section of the brain; these cells emit abnormal, recurring, uncontrolled electrical discharges. Seizures are not caused by changes in blood flow or electrolytes.

26. The nurse planning the care of a patient with head injuries is addressing the patient's nursing diagnosis of "sleep deprivation." What action should the nurse implement? A) Administer a benzodiazepine at bedtime each night. B) Do not disturb the patient between 2200 and 0600. C) Cluster overnight nursing activities to minimize disturbances. D) Ensure that the patient does not sleep during the day.

C Feedback: To allow the patient longer times of uninterrupted sleep and rest, the nurse can group nursing care activities so that the patient is disturbed less frequently. However, it is impractical and unsafe to provide no care for an 8-hour period. The use of benzodiazepines should be avoided.

35. The nurse caring for a patient with a spinal cord injury notes that the patient is exhibiting early signs and symptoms of disuse syndrome. Which of the following is the most appropriate nursing action? A) Limit the amount of assistance provided with ADLs. B) Collaborate with the physical therapist and immobilize the patient's extremities temporarily. C) Increase the frequency of ROM exercises. D) Educate the patient about the importance of frequent position changes.

C Feedback: To prevent disuse syndrome, ROM exercises must be provided at least four times a day, and care is taken to stretch the Achilles tendon with exercises. The patient is repositioned frequently and is maintained in proper body alignment whether in bed or in a wheelchair. The patient must be repositioned by caregivers, not just taught about repositioning. It is inappropriate to limit assistance for the sole purpose of preventing disuse syndrome.

32. A patient with a spinal cord injury has experienced several hypotensive episodes. How can the nurse best address the patient's risk for orthostatic hypotension? A) Administer an IV bolus of normal saline prior to repositioning. B) Maintain bed rest until normal BP regulation returns. C) Monitor the patient's BP before and during position changes. D) Allow the patient to initiate repositioning.

C Feedback: To prevent hypotensive episodes, close monitoring of vital signs before and during position changes is essential. Prolonged bed rest carries numerous risks and it is not possible to provide a bolus before each position change. Following the patient's lead may or may not help regulate BP.

39. A nurse is collaborating with the interdisciplinary team to help manage a patient's recurrent headaches. What aspect of the patient's health history should the nurse identify as a potential contributor to the patient's headaches? A) The patient leads a sedentary lifestyle. B) The patient takes vitamin D and calcium supplements. C) The patient takes vasodilators for the treatment of angina. D) The patient has a pattern of weight loss followed by weight gain.

C Feedback: Vasodilators are known to contribute to headaches. Weight fluctuations, sedentary lifestyle, and vitamin supplements are not known to have this effect.

24. A patient who has been experiencing numerous episodes of unexplained headaches and vomiting has subsequently been referred for testing to rule out a brain tumor. What characteristic of the patient's vomiting is most consistent with a brain tumor? A) The patient's vomiting is accompanied by epistaxis. B) The patient's vomiting does not relieve his nausea. C) The patient's vomiting is unrelated to food intake. D) The patient's emesis is blood-tinged.

C Feedback: Vomiting is often unrelated to food intake if caused by a brain tumor. The presence or absence of blood is not related to the possible etiology and vomiting may or may not relieve the patient's nausea.

The nurse is discharging a patient home after surgery for trigeminal neuralgia. What advice should the nurse provide to this patient in order to reduce the risk of injury? A) Avoid watching television or using a computer for more than 1 hour at a time. B) Use OTC antibiotic eye drops for at least 14 days. C) Avoid rubbing the eye on the affected side of the face. D) Rinse the eye on the affected side with normal saline daily for 1 week.

C) Avoid rubbing the eye on the affected side of the face. If the surgery results in sensory deficits to the affected side of the face, the patient is instructed not to rub the eye because the pain of a resulting injury will not be detected. There is no need to limit TV viewing or to rinse the eye daily. Antibiotics may or may not be prescribed, and these would not reduce the risk of injury.

A 33-year-old patient presents at the clinic with complaints of weakness, incoordination, dizziness, and loss of balance. The patient is hospitalized and diagnosed with MS. What sign or symptom, revealed during the initial assessment, is typical of MS? A) Diplopia, history of increased fatigue, and decreased or absent deep tendon reflexes B) Flexor spasm, clonus, and negative Babinskis reflex C) Blurred vision, intention tremor, and urinary hesitancy D) Hyperactive abdominal reflexes and history of unsteady gait and episodic paresthesia in both legs

C) Blurred vision, intention tremor, and urinary hesitancy Optic neuritis, leading to blurred vision, is a common early sign of MS, as is intention tremor (tremor when performing an activity). Nerve damage can cause urinary hesitancy. In MS, deep tendon reflexes are increased or hyperactive. A positive Babinskis reflex is found in MS. Abdominal reflexes are absent with MS.

A patient is admitted through the ED with suspected St. Louis encephalitis. The unique clinical feature of St. Louis encephalitis will make what nursing action a priority? A) Serial assessments of hemoglobin levels B) Blood glucose monitoring C) Close monitoring of fluid balance D) Assessment of pain along dermatomes

C) Close monitoring of fluid balance A unique clinical feature of St. Louis encephalitis is SIADH with hyponatremia. As such, it is important to monitor the patients intake and output closely.

A patient is being admitted to the neurologic ICU with suspected herpes simplex virus encephalitis. What nursing action best addresses the patients complaints of headache? A) Initiating a patient-controlled analgesia (PCA) of morphine sulfate B) Administering hydromorphone (Dilaudid) IV as needed C) Dimming the lights and reducing stimulation D) Distracting the patient with activity

C) Dimming the lights and reducing stimulation Comfort measures to reduce headache include dimming the lights, limiting noise and visitors, grouping nursing interventions, and administering analgesic agents. Opioid analgesic medications may mask neurologic symptoms; therefore, they are used cautiously. Non-opioid analgesics may be preferred. Distraction is unlikely to be effective, and may exacerbate the patients pain.

A nurse is planning the care of a 28-year-old woman hospitalized with a diagnosis of myasthenia gravis. What approach would be most appropriate for the care and scheduling of diagnostic procedures for this patient? A) All at one time, to provide a longer rest period B) Before meals, to stimulate her appetite C) In the morning, with frequent rest periods D) Before bedtime, to promote rest

C) In the morning, with frequent rest periods Procedures should be spaced to allow for rest in between. Procedures should be avoided before meals, or the patient may be too exhausted to eat. Procedures should be avoided near bedtime if possible.

The critical care nurse is caring for 25-year-old man admitted to the ICU with a brain abscess. What is a priority nursing responsibility in the care of this patient? A) Maintaining the patients functional independence B) Providing health education C) Monitoring neurologic status closely D) Promoting mobility

C) Monitoring neurologic status closely Vigilant neurologic monitoring is a key aspect of caring for a patient who has a brain abscess. This supersedes education, ADLs, and mobility, even though these are all valid and important aspects of nursing care.

A patient with Guillain-Barr syndrome has experienced a sharp decline in vital capacity. What is the nurses most appropriate action? A) Administer bronchodilators as ordered. B) Remind the patient of the importance of deep breathing and coughing exercises. C) Prepare to assist with intubation. D) Administer supplementary oxygen by nasal cannula.

C) Prepare to assist with intubation. For the patient with Guillain-Barr syndrome, mechanical ventilation is required if the vital capacity falls, making spontaneous breathing impossible and tissue oxygenation inadequate. Each of the other listed actions is likely insufficient to meet the patients oxygenation needs.

The critical care nurse is admitting a patient in myasthenic crisis to the ICU. The nurse should prioritize what nursing action in the immediate care of this patient? A) Suctioning secretions B) Facilitating ABG analysis C) Providing ventilatory assistance D) Administering tube feedings

C) Providing ventilatory assistance Providing ventilatory assistance takes precedence in the immediate management of the patient with myasthenic crisis. It may be necessary to suction secretions and/or provide tube feedings, but they are not the priority for this patient. ABG analysis will be done, but this is not the priority.

A 48-year-old patient has been diagnosed with trigeminal neuralgia following recent episodes of unilateral face pain. The nurse should recognize what implication of this diagnosis? A) The patient will likely require lifelong treatment with anticholinergic medications. B) The patient has a disproportionate risk of developing myasthenia gravis later in life.' C) The patient needs to be assessed for MS. D) The disease is self-limiting and the patient will achieve pain relief over time.

C) The patient needs to be assessed for MS. Patients that develop trigeminal neuralgia before age 50 should be evaluated for the coexistent of MS because trigeminal neuralgia occurs in approximately 5% of patients with MS. Treatment does not include anticholinergics and the disease is not self-limiting. Trigeminal neuralgia is not associated with an increased risk of myasthenia gravis.

21. The nurse educator is discussing neoplasms with a group of recent graduates. The educator explains that the effects of neoplasms are caused by the compression and infiltration of normal tissue. The physiologic changes that result can cause what pathophysiologic events? Select all that apply. A) Intracranial hemorrhage B) Infection of cerebrospinal fluid C) Increased ICP D) Focal neurologic signs E) Altered pituitary function

C, D, E Feedback: The effects of neoplasms are caused by the compression and infiltration of tissue. A variety of physiologic changes result, causing any or all of the following pathophysiologic events: increased ICP and cerebral edema, seizure activity and focal neurologic signs, hydrocephalus, and altered pituitary function.

A nurse is teaching a client who has Parkinson's disease and is prescribed carbidopa-levodopa. Which of the following client statements indicates an understanding of the teaching? A. "I should expect an increase in my blood pressure while taking this medication" B. "I should take this medication 2 hr after meals to increase absorptions" C. "I should expect that this medication can cause me to be drowsy" D. "I should expect this medication to be effective within 48 hrs"

C. "I should expect that this medication can cause me to be drowsy." Drowsiness is a known adverse effect of carbidopa-levodopa; therefore, clients are taught to avoid heavy machinery and driving if they experience drowsiness. "I should expect an increase in my blood pressure while taking this medication."Orthostatic hypotension is an adverse effect of carbidopa-levodopa. "I should take this medication 2 hours after meals to increase absorption."-Carbidopa-levodopa should be administered before meals to increase absorption and transport the medication across the blood-brain barrier. "I should expect this medication to be effective within 48 hours." The nurse should inform the client that the medication can take 2 to 3 months to take effect.

According to question 16, the patient's blood pressure is 130/88. What is the patient's mean arterial pressure (MAP)?* A. 42 B. 74 C. 102 D. 88

C. 102 The answer is C. MAP is calculated by taking the DBP (88) and multiplying it by 2. This equals 176. Then take this number and add the SBP (130). This equals 306. Then take this number and divide by 3, which equal 102.

Which patient below with ICP is experiencing Cushing's Triad? A patient with the following:* A. BP 150/112, HR 110, RR 8 B. BP 90/60, HR 80, RR 22 C. BP 200/60, HR 50, RR 8 D. BP 80/40, HR 49, RR 12

C. BP 200/60, HR 50, RR 8 The answer is C. These vital signs represent Cushing's triad. There is an increase in the systolic pressure, widening pulse pressure of 140 (200-60=140), bradycardia, and bradypnea.

A nurse is caring for a client who has viral meningitis. Which of the following actions should the nurse take? A. Assess the client's neurologic status every 8 hr B. Initiate droplet precautions C. Check capillary refill at least every 4 hr D. Place the client in a well-lit environment

C. Check capillary refill at least every 4hr-The nurse should perform a complete vascular assessment at least every 4 hr to monitor for vascular compromise Place the client in a well-lit environment. The nurse should minimize the client's exposure to light from windows and overhead lights because photophobia, or light sensitivity, is a manifestation of viral meningitis. Assess the client's neurologic status every 8 hr.The nurse should assess the client's vital signs and neurologic status at least every 2 to 4 hr. Initiate droplet precautions.-The nurse should implement droplet precautions for clients who have bacterial meningitis. Standard precautions are sufficient for clients who have viral meningitis.

A nurse is planning care for a client following a lumbar puncture. Which of the following actions should the nurse plan to take? A. Apply pressure dressing to the site for 8 hr B. Restrict the client's fluid intake for 24 hr C. Ensure that the client lies flat for up to 12 hr D. Inform the client that neck stiffness is an expected outcome of the procedure

C. Ensure that the client lies flat for up to 12 hr.-The client should lie flat for up to 12 hr to prevent cerebrospinal fluid leakage from the puncture site, which can cause a headache Apply a pressure dressing to the site for 8 hr.The nurse should apply pressure to the site and then apply an adhesive bandage, not a pressure dressing. Restrict the client's fluid intake for 24 hr.The client should increase fluid intake to replace the cerebrospinal fluid the provider removed during the procedure. Inform the client that neck stiffness is an expected outcome of the procedure. The nurse should instruct the client to report complications of a lumber puncture such as voiding difficulties, fever, stiffness of the back or neck, nausea, and vomiting.

A nurse is caring for a client who has a retinal detachment. which of the following findings should the nurse expect? A. Photophobia B. Complete vision loss C. Flashes of bright light D. Cloudiness of the lends

C. Flashes of bright light The nurse should expect a client who has a retinal detachment to see flashes of bright light or floating dark spots in the affected eye as the retinal layers separate. Photophobia - The nurse should expect photophobia in a client who has a migraine headache. Complete vision loss -The nurse should expect a client who has a retinal detachment to have some visual field loss in the area of the detachment, but complete vision loss is not an expected finding. Cloudiness of the lens- The nurse should expect a client who has cataracts to experience cloudiness of the lens

A patient has a ventriculostomy. Which finding would you immediately report to the doctor?* A. Temperature 98.4 'F B. CPP 70 mmHg C. ICP 24 mmHg

C. ICP 24 mmHg The answer is C. A ventriculostomy is a catheter inserted in the area of the lateral ventricle to assess ICP. It will help drain CSF during increase pressure readings and measure ICP. The nurse must monitor for ICP levels greater than 20 mmHg and report it to the doctor.

A nurse is assessing a client who had a right hemisphere stroke. Which of the following neurologic deficits should the nurse expect? A. Aphasia B. Right-sided neglect C. Impulsive behavior D. Inability to read

C. Impulsive behavior- The nurse should expect clients who had a right hemispheric stroke to demonstrate impulsive behavior, poor judgment, and lack of awareness of neurologic deficits. AphasiaClients who had a left hemispheric stroke are likely to have aphasia. Right-sided neglectClients who had a right hemispheric stroke are likely to have neurologic deficits on the left side of the body, not the right side. The nurse should expect the client to be unaware of and unable to move the left side of the body. Inability to read Clients who had a left hemispheric stroke are likely to have difficulty reading due to the inability to discriminate different letters and words.

A nurse is caring for a client who has multiple sclerosis. Which of the following should the nurse expect? A. Hypoactive deep-tendon reflexes B. Ascending paralysis C. Intention tremors D. Increased lacrimation

C. Intention tremors- Clients who have multiple sclerosis are at risk for motor dysfunction, with intention tremors, poor coordination, and loss of balance. Hypoactive deep-tendon reflexes- Clients who have multiple sclerosis have hyperactive deep-tendon reflexes. Ascending paralysis- Clients who have Guillain-Barré syndrome are at risk for ascending paralysis. Increased lacrimation-Increased lacrimation, or tearing of the eyes, is an expected finding of myasthenia gravis during a cholinergic crisis.

A nurse is caring for a client who has advancing amyotrophic lateral sclerosis. Which of the following interventions is the nurse's priority? A. Provide frequent rest period throughout the day B. Administer pain medication on a regular schedule C. Monitor pulse oximetry findings D. Administer baclofen for spasticity

C. Monitor pulse oximetry findings.- The greatest risk to the client is respiratory compromise due to progressive paralysis of respiratory muscles. Therefore, the priority intervention is to monitor the client's oxygen saturation to identify respiratory compromise as soon as possible. Provide frequent rest periods throughout the day.The nurse should provide frequent rest periods throughout the day because the client's fatigue will increase as the disease progresses. However, this is not the priority intervention. Administer pain medication on a regular schedule.The nurse should administer pain medication on a regular schedule to keep the client's pain level under control. However, this is not the priority intervention. Administer baclofen for spasticity. The nurse should administer baclofen to manage spasticity that can interfere with self-care. However, this is not the priority intervention

A patient with increased ICP has the following vital signs: blood pressure 99/60, HR 65, Temperature 101.6 'F, respirations 14, oxygen saturation of 95%. ICP reading is 21 mmHg. Based on these findings you would?* A. Administered PRN dose of a vasopressor B. Administer 2 L of oxygen C. Remove extra blankets and give the patient a cool bath D. Perform suctioning

C. Remove extra blankets and give the patient a cool bath The answer is C. It is important to monitor the patient for hyperthermia (a fever). A fever increases ICP and cerebral blood volume, and metabolic needs of the patient. The nurse can administer antipyretics per MD order, remove extra blankets, decrease room temperature, give a cool bath or use a cooling system. Remember it is important to prevent shivering (this also increases metabolic needs and ICP).

A nurse is caring for a client who is 72 hr postoperative following an above-the-knee amputation and reports phantom limb pain. Which of the following actions should the nurse take? A. Remind the client that the surgery removed the limb B. Change the dressing on the client's residual limb C. Request a prescription for gabapentin for the client D. Elevate the client's residual limb above the heart level

C. Request a prescription for gabapentin for the client. The nurse should request a prescription for a nonopioid medication to help minimize phantom limb pain. Gabapentin is an oral antiepileptic medication that is effective for treating sharp, burning, phantom limb pain. Remind the client that the surgery removed the limb.-It is not therapeutic for the nurse to remind the client that the limb is gone because it does not address the client's pain. Change the dressing on the client's residual limb.Changing the dressing on the client's residual limb does not address the client's pain. Elevate the client's residual limb above heart level. The nurse should only elevate the client's residual limb above the heart level within the first 48 hr following the surgery. After that time, doing so can cause a hip or knee flexion contracture.

A patient who experienced a cerebral hemorrhage is at risk for developing increased ICP. Which sign and symptom below is the EARLIEST indicator the patient is having this complication?* A. Bradycardia B. Decerebrate posturing C. Restlessness D. Unequal pupil size

C. Restlessness The answer is C. Mental status changes are the earliest indicator a patient is experiencing increased ICP. All the other signs and symptoms listed happen later.

A patient is experiencing hyperventilation and has a PaCO2 level of 52. The patient has an ICP of 20 mmHg. As the nurse you know that the PaCO2 level will?* A. cause vasoconstriction and decrease the ICP B. promote diuresis and decrease the ICP C. cause vasodilation and increase the ICP D. cause vasodilation and decrease the ICP

C. cause vasodilation and increase the ICP. The answer is C. An elevated carbon dioxide level (52 is high...normal 35-45) in the blood will cause vasodilation (NOT constriction), which will increase ICP (normal ICP 5 to 15 mmHg). Therefore, many patients with severe ICP may need to be mechanical ventilated so PaCO2 levels can be lowered (30-35), which will lead to vasoconstriction and decrease ICP (with constriction there is less blood volume and flow going to the brain and this helps decrease pressure)....remember Monro-Kellie hypothesis.

A client with a traumatic brain injury from a motor vehicle crash is monitored for signs of increased intracranial pressure (ICP). Which sign does the nurse monitor for?

Changes in breathing pattern Changes in breathing pattern may cause hypoxia and hypercapnia, which can increase ICP. Dizziness is indicative of brain injury. Increasing level of consciousness and reactive pupils are desired outcomes for this client.

A client recovering from a stroke reports double vision that is preventing the client from effectively completing activities of daily living. How does the nurse help the client compensate?

Covers the affected eye Covering the client's eye with a patch prevents diplopia. The client who is recovering from a stroke should always be approached on the unaffected side. The nurse may encourage side-to-side head turning for clients with hemianopsia (blindness in half of the visual field). Objects should be placed in the field of vision for the client with a decreased visual field.

A nurse is assessing a client who has a new diagnosis of osteoarthritis. Which of the following findings should the nurse expect? (Select all that apply) A. Crepitus with joint movement B. Decreased range of motion of the affected joint C. Low-grade fever D. Spongy tissue over the joints E. Joint pain that resolves with rest

Crepitus with joint movement is correct. Osteoarthritis is a degenerative joint disease. Crepitus, a grating sound, is an expected finding with clients who have osteoarthritis as loosened bone and cartilage move around in the fluid inside the joint. Decreased range of motion of the affected joint is correct. Decreased range of motion is an expected finding with clients who have osteoarthritis because the client's pain limits movement. Low-grade fever is incorrect. Osteoarthritis does not cause systemic manifestations. Rheumatoid arthritis causes many systemic manifestations, including low-grade fever, weakness, anorexia, and paresthesias. Spongy tissue over the joints is incorrect. Spongy joint tissue is an expected finding with rheumatoid arthritis, which is an inflammatory disease, not a degenerative disease. Joint pain that resolves with rest is correct. Joint pain that resolves with rest is an expected finding with clients who have osteoarthritis. A client who has osteoarthritis experiences increased pain with activity and decreased pain with rest.

11. A patient who suffered a spinal cord injury is experiencing an exaggerated autonomic response. What aspect of the patient's current health status is most likely to have precipitated this event? A) The patient received a blood transfusion. B) The patient's analgesia regimen was recent changed. C) The patient was not repositioned during the night shift. D) The patient's urinary catheter became occluded.

D Feedback: A distended bladder is the most common cause of autonomic dysreflexia. Infrequent positioning is a less likely cause, although pressure ulcers or tactile stimulation can cause it. Changes in mediations or blood transfusions are unlikely causes.

15. A patient diagnosed with a cerebral aneurysm reports a severe headache to the nurse. What action is a priority for the nurse? A) Sit with the patient for a few minutes. B) Administer an analgesic. C) Inform the nurse-manager. D) Call the physician immediately.

D Feedback: A headache may be an indication that the aneurysm is leaking. The nurse should notify the physician immediately. The physician will decide whether administration of an analgesic is indicated. Informing the nurse-manager is not necessary. Sitting with the patient is appropriate, once the physician has been notified of the change in the patient's condition.

9. While assessing the patient at the beginning of the shift, the nurse inspects a surgical dressing covering the operative site after the patients' cervical diskectomy. The nurse notes that the drainage is 75% saturated with serosanguineous discharge. What is the nurse's most appropriate action? A) Page the physician and report this sign of infection. B) Reinforce the dressing and reassess in 1 to 2 hours. C) Reposition the patient to prevent further hemorrhage. D) Inform the surgeon of the possibility of a dural leak.

D Feedback: After a cervical diskectomy, the nurse will monitor the operative site and dressing covering this site. Serosanguineous drainage may indicate a dural leak. This constitutes a risk for meningitis, but is not a direct sign of infection. This should be reported to the surgeon, not just reinforced and observed.

5. A patient with a documented history of seizure disorder experiences a generalized seizure. What nursing action is most appropriate? A) Restrain the patient to prevent injury. B) Open the patient's jaws to insert an oral airway. C) Place patient in high Fowler's position. D) Loosen the patient's restrictive clothing.

D Feedback: An appropriate nursing intervention would include loosening any restrictive clothing on the patient. No attempt should be made to restrain the patient during the seizure because muscular contractions are strong and restraint can produce injury. Do not attempt to pry open jaws that are clenched in a spasm to insert anything. Broken teeth and injury to the lips and tongue may result from such an action. If possible, place the patient on one side with head flexed forward, which allows the tongue to fall forward and facilitates drainage of saliva and mucus.

8. The nurse is caring for a patient with increased intracranial pressure (ICP). The patient has a nursing diagnosis of "ineffective cerebral tissue perfusion." What would be an expected outcome that the nurse would document for this diagnosis? A) Copes with sensory deprivation. B) Registers normal body temperature. C) Pays attention to grooming. D) Obeys commands with appropriate motor responses.

D Feedback: An expected outcome of the diagnosis of ineffective cerebral tissue perfusion in a patient with increased intracranial pressure (ICP) would include obeying commands with appropriate motor responses. Vitals signs and neurologic status are assessed every 15 minutes to every hour. Coping with sensory deprivation would relate to the nursing diagnosis of "disturbed sensory perception." The outcome of "registers normal body temperature" relates to the diagnosis of "potential for ineffective thermoregulation." Body image disturbance would have a potential outcome of "pays attention to grooming."

36. The nurse is caring for a patient who has undergone supratentorial removal of a pituitary mass. What medication would the nurse expect to administer prophylactically to prevent seizures in this patient? A) Prednisone B) Dexamethasone C) Cafergot D) Phenytoin

D Feedback: Antiseizure medication (phenytoin, diazepam) is often prescribed prophylactically for patients who have undergone supratentorial craniotomy because of the high risk of seizures after this procedure. Prednisone and dexamethasone are steroids and do not prevent seizures. Cafergot is used in the treatment of migraines.

22. The nurse is providing care for a patient who is withdrawing from heavy alcohol use. The nurse and other members of the care team are present at the bedside when the patient has a seizure. In preparation for documenting this clinical event, the nurse should note which of the following? A) The ability of the patient to follow instructions during the seizure. B) The success or failure of the care team to physically restrain the patient. C) The patient's ability to explain his seizure during the postictal period. D) The patient's activities immediately prior to the seizure.

D Feedback: Before and during a seizure, the nurse observes the circumstances before the seizure, including visual, auditory, or olfactory stimuli; tactile stimuli; emotional or psychological disturbances; sleep; and hyperventilation. Communication with the patient is not possible during a seizure and physical restraint is not attempted. The patient's ability to explain the seizure is not clinically relevant.

18. What should the nurse suspect when hourly assessment of urine output on a patient postcraniotomy exhibits a urine output from a catheter of 1,500 mL for two consecutive hours? A) Cushing syndrome B) Syndrome of inappropriate antidiuretic hormone (SIADH) C) Adrenal crisis D) Diabetes insipidus

D Feedback: Diabetes insipidus is an abrupt onset of extreme polyuria that commonly occurs in patients after brain surgery. Cushing syndrome is excessive glucocorticoid secretion resulting in sodium and water retention. SIADH is the result of increased secretion of ADH; the patient becomes volume-overloaded, urine output diminishes, and serum sodium concentration becomes dilute. Adrenal crisis is undersecretion of glucocorticoids resulting in profound hypoglycemia, hypovolemia, and hypotension.

7. A 37-year-old man is brought to the clinic by his wife because he is experiencing loss of motor function and sensation. The physician suspects the patient has a spinal cord tumor and hospitalizes him for diagnostic testing. In light of the need to diagnose spinal cord compression from a tumor, the nurse will most likely prepare the patient for what test? A) Anterior-posterior x-ray B) Ultrasound C) Lumbar puncture D) MRI

D Feedback: The MRI scan is the most commonly used diagnostic procedure. It is the most sensitive diagnostic tool that is particularly helpful in detecting epidural spinal cord compression and vertebral bone metastases.

27. The nurse has implemented interventions aimed at facilitating family coping in the care of a patient with a traumatic brain injury. How can the nurse best facilitate family coping? A) Help the family understand that the patient could have died. B) Emphasize the importance of accepting the patient's new limitations. C) Have the members of the family plan the patient's inpatient care. D) Assist the family in setting appropriate short-term goals.

D Feedback: Helpful interventions to facilitate coping include providing family members with accurate and honest information and encouraging them to continue to set well-defined, short-term goals. Stating that a patient's condition could be worse downplays their concerns. Emphasizing the importance of acceptance may not necessarily help the family accept the patient's condition. Family members cannot normally plan a patient's hospital care, although they may contribute to the care in some ways.

23. A 13-year-old was brought to the ED, unconscious, after being hit in the head by a baseball. When the child regains consciousness, 5 hours after being admitted, he cannot remember the traumatic event. MRI shows no structural sign of injury. What injury would the nurse suspect the patient has? A) Diffuse axonal injury B) Grade 1 concussion with frontal lobe involvement C) Contusion D) Grade 3 concussion with temporal lobe involvement

D Feedback: In a grade 3 concussion there is a loss of consciousness lasting from seconds to minutes. Temporal lobe involvement results in amnesia. Frontal lobe involvement can cause uncharacteristic behavior and a grade 1 concussion does not involve loss of consciousness. Diagnostic studies may show no apparent structural sign of injury, but the duration of unconsciousness is an indicator of the severity of the concussion. Diffuse axonal injury (DAI) results from widespread shearing and rotational forces that produce damage throughout the brain—to axons in the cerebral hemispheres, corpus callosum, and brain stem. In cerebral contusion, a moderate to severe head injury, the brain is bruised and damaged in a specific area because of severe acceleration-deceleration force or blunt trauma.

11. A patient has just been diagnosed with Parkinson's disease and the nurse is planning the patient's subsequent care for the home setting. What nursing diagnosis should the nurse address when educating the patient's family? A) Risk for infection B) Impaired spontaneous ventilation C) Unilateral neglect D) Risk for injury

D Feedback: Individuals with Parkinson's disease face a significant risk for injury related to the effects of dyskinesia. Unilateral neglect is not characteristic of the disease, which affects both sides of the body. Parkinson's disease does not directly constitute a risk for infection or impaired respiration.

27. A patient has a poor prognosis after being involved in a motor vehicle accident resulting in a head injury. As the patient's ICP increases and condition worsens, the nurse knows to assess for indications of approaching death. These indications include which of the following? A) Hemiplegia B) Dry mucous membranes C) Signs of internal bleeding D) Loss of brain stem reflexes

D Feedback: Loss of brain stem reflexes, including pupillary, corneal, gag, and swallowing reflexes, is an ominous sign of approaching death. Dry mucous membranes, hemiplegia, and bleeding must be promptly addressed, but none of these is a common sign of impending death.

10. A patient, diagnosed with cancer of the lung, has just been told he has metastases to the brain. What change in health status would the nurse attribute to the patient's metastatic brain disease? A) Chronic pain B) Respiratory distress C) Fixed pupils D) Personality changes

D Feedback: Neurologic signs and symptoms include headache, gait disturbances, visual impairment, personality changes, altered mentation (memory loss and confusion), focal weakness, paralysis, aphasia, and seizures. Pain, respiratory distress, and fixed pupils are not among the more common neurologic signs and symptoms of metastatic brain disease.

12. The nurse is caring for a patient with Huntington disease who has been admitted to the hospital for treatment of malnutrition. What independent nursing action should be implemented in the patient's plan of care? A) Firmly redirect the patient's head when feeding. B) Administer phenothiazines after each meal as ordered. C) Encourage the patient to keep his or her feeding area clean. D) Apply deep, gentle pressure around the patient's mouth to aid swallowing.

D Feedback: Nursing interventions for a patient who has inadequate nutritional intake should include the following: Apply deep gentle pressure around the patient's mouth to assist with swallowing, and administer phenothiazines prior to the patient's meal as ordered. The nurse should disregard the mess of the feeding area and treat the person with dignity. Stiffness and turning away by the patient during feeding are uncontrollable choreiform movements and should not be interrupted.

33. A nurse on the neurologic unit is providing care for a patient who has spinal cord injury at the level of C4. When planning the patient's care, what aspect of the patient's neurologic and functional status should the nurse consider? A) The patient will be unable to use a wheelchair. B) The patient will be unable to swallow food. C) The patient will be continent of urine, but incontinent of bowel. D) The patient will require full assistance for all aspects of elimination.

D Feedback: Patients with a lesion at C4 are fully dependent for elimination. The patient is dependent for feeding, but is able to swallow. The patient will be capable of using an electric wheelchair.

19. During the examination of an unconscious patient, the nurse observes that the patient's pupils are fixed and dilated. What is the most plausible clinical significance of the nurse's finding? A) It suggests onset of metabolic problems. B) It indicates paralysis on the right side of the body. C) It indicates paralysis of cranial nerve X. D) It indicates an injury at the midbrain level.

D Feedback: Pupils that are fixed and dilated indicate injury at the midbrain level. This finding is not suggestive of unilateral paralysis, metabolic deficits, or damage to CN X.

21. The public health nurse is planning a health promotion campaign that reflects current epidemiologic trends. The nurse should know that hemorrhagic stroke currently accounts for what percentage of total strokes in the United States? A) 43% B) 33% C) 23% D) 13%

D Feedback: Strokes can be divided into two major categories: ischemic (87%), in which vascular occlusion and significant hypoperfusion occur, and hemorrhagic (13%), in which there is extravasation of blood into the brain or subarachnoid space.

19. What should be included in the patient's care plan when establishing an exercise program for a patient affected by a stroke? A) Schedule passive range of motion every other day. B) Keep activity limited, as the patient may be over stimulated. C) Have the patient perform active range-of-motion (ROM) exercises once a day. D) Exercise the affected extremities passively four or five times a day.

D Feedback: The affected extremities are exercised passively and put through a full ROM four or five times a day to maintain joint mobility, regain motor control, prevent development of a contracture in the paralyzed extremity, prevent further deterioration of the neuromuscular system, and enhance circulation. Active ROM exercises should ideally be performed more than once per day.

32. A patient who was diagnosed with Parkinson's disease several months ago recently began treatment with levodopa-carbidopa. The patient and his family are excited that he has experienced significant symptom relief. The nurse should be aware of what implication of the patient's medication regimen? A) The patient is in a "honeymoon period" when adverse effects of levodopa-carbidopa are not yet evident. B) Benefits of levodopa-carbidopa do not peak until 6 to 9 months after the initiation of treatment. C) The patient's temporary improvement in status is likely unrelated to levodopa-carbidopa. D) Benefits of levodopa-carbidopa often diminish after 1 or 2 years of treatment.

D Feedback: The beneficial effects of levodopa therapy are most pronounced in the first year or two of treatment. Benefits begin to wane and adverse effects become more severe over time. However, a "honeymoon period" of treatment is not known.

7. An ED nurse has just received a call from EMS that they are transporting a 17-year-old man who has just sustained a spinal cord injury (SCI). The nurse recognizes that the most common cause of this type of injury is what? A) Sports-related injuries B) Acts of violence C) Injuries due to a fall D) Motor vehicle accidents

D Feedback: The most common causes of SCIs are motor vehicle crashes (46%), falls (22%), violence (16%), and sports (12%).

15. A male patient with a metastatic brain tumor is having a generalized seizure and begins vomiting. What should the nurse do first? A) Perform oral suctioning. B) Page the physician. C) Insert a tongue depressor into the patient's mouth. D) Turn the patient on his side.

D Feedback: The nurse's first response should be to place the patient on his side to prevent him from aspirating emesis. Inserting something into the seizing patient's mouth is no longer part of a seizure protocol. Obtaining supplies to suction the patient would be a delegated task. Paging or calling the physician would only be necessary if this is the patient's first seizure.

7. A patient with a cerebral aneurysm exhibits signs and symptoms of an increase in intracranial pressure (ICP). What nursing intervention would be most appropriate for this patient? A) Range-of-motion exercises to prevent contractures B) Encouraging independence with ADLs to promote recovery C) Early initiation of physical therapy D) Absolute bed rest in a quiet, nonstimulating environment

D Feedback: The patient is placed on immediate and absolute bed rest in a quiet, nonstressful environment because activity, pain, and anxiety elevate BP, which increases the risk for bleeding. Visitors are restricted. The nurse administers all personal care. The patient is fed and bathed to prevent any exertion that might raise BP.

25. After a subarachnoid hemorrhage, the patient's laboratory results indicate a serum sodium level of less than 126 mEq/L. What is the nurse's most appropriate action? A) Administer a bolus of normal saline as ordered. B) Prepare the patient for thrombolytic therapy as ordered. C) Facilitate testing for hypothalamic dysfunction. D) Prepare to administer 3% NaCl by IV as ordered.

D Feedback: The patient may be experiencing syndrome of inappropriate antidiuretic hormone (SIADH) or cerebral salt-wasting syndrome. The treatment most often is the use of IV hypertonic 3% saline. A normal saline bolus would exacerbate the problem and there is no indication for tests of hypothalamic function or thrombolytic therapy.

37. A nursing student is writing a care plan for a newly admitted patient who has been diagnosed with a stroke. What major nursing diagnosis should most likely be included in the patient's plan of care? A) Adult failure to thrive B) Post-trauma syndrome C) Hyperthermia D) Disturbed sensory perception

D Feedback: The patient who has experienced a stroke is at a high risk for disturbed sensory perception. Stroke is associated with multiple other nursing diagnoses, but hyperthermia, adult failure to thrive, and post-trauma syndrome are not among these.

20. A female patient is diagnosed with a right-sided stroke. The patient is now experiencing hemianopsia. How might the nurse help the patient manage her potential sensory and perceptional difficulties? A) Keep the lighting in the patient's room low. B) Place the patient's clock on the affected side. C) Approach the patient on the side where vision is impaired. D) Place the patient's extremities where she can see them.

D Feedback: The patient with homonymous hemianopsia (loss of half of the visual field) turns away from the affected side of the body and tends to neglect that side and the space on that side; this is called amorphosynthesis. In such instances, the patient cannot see food on half of the tray, and only half of the room is visible. It is important for the nurse to remind the patient constantly of the other side of the body, to maintain alignment of the extremities, and if possible, to place the extremities where the patient can see them. Patients with a decreased field of vision should be approached on the side where visual perception is intact. All visual stimuli (clock, calendar, and television) should be placed on this side. The patient can be taught to turn the head in the direction of the defective visual field to compensate for this loss. Increasing the natural or artificial lighting in the room and providing eyeglasses are important in increasing vision. There is no reason to keep the lights dim.

8. A patient recovering from a stroke has severe shoulder pain from subluxation of the shoulder and is being cared for on the unit. To prevent further injury and pain, the nurse caring for this patient is aware of what principle of care? A) The patient should be fitted with a cast because use of a sling should be avoided due to adduction of the affected shoulder. B) Elevation of the arm and hand can lead to further complications associated with edema. C) Passively exercising the affected extremity is avoided in order to minimize pain. D) The patient should be taught to interlace fingers, place palms together, and slowly bring scapulae forward to avoid excessive force to shoulder.

D Feedback: To prevent shoulder pain, the nurse should never lift a patient by the flaccid shoulder or pull on the affected arm or shoulder. The patient is taught how to move and exercise the affected arm/shoulder through proper movement and positioning. The patient is instructed to interlace the fingers, place the palms together, and push the clasped hands slowly forward to bring the scapulae forward; he or she then raises both hands above the head. This is repeated throughout the day. The use of a properly worn sling when the patient is out of bed prevents the paralyzed upper extremity from dangling without support. Range-of-motion exercises are still vitally important in preventing a frozen shoulder and ultimately atrophy of subcutaneous tissues, which can cause more pain. Elevation of the arm and hand is also important in preventing dependent edema of the hand.

13. A patient with a head injury has been increasingly agitated and the nurse has consequently identified a risk for injury. What is the nurse's best intervention for preventing injury? A) Restrain the patient as ordered. B) Administer opioids PRN as ordered. C) Arrange for friends and family members to sit with the patient. D) Pad the side rails of the patient's bed.

D Feedback: To protect the patient from self-injury, the nurse uses padded side rails. The nurse should avoid restraints, because straining against them can increase ICP or cause other injury. Narcotics used to control restless patients should be avoided because these medications can depress respiration, constrict the pupils, and alter the patient's responsiveness. Visitors should be limited if the patient is agitated.

21. The ED is notified that a 6-year-old is in transit with a suspected brain injury after being struck by a car. The child is unresponsive at this time, but vital signs are within acceptable limits. What will be the primary goal of initial therapy? A) Promoting adequate circulation B) Treating the child's increased ICP C) Assessing secondary brain injury D) Preserving brain homeostasis

D Feedback: All therapy is directed toward preserving brain homeostasis and preventing secondary brain injury, which is injury to the brain that occurs after the original traumatic event. The scenario does not indicate the child has increased ICP or a secondary brain injury at this point. Promoting circulation is likely secondary to the broader goal of preserving brain homeostasis.

The nurse is preparing to provide care for a patient diagnosed with myasthenia gravis. The nurse should know that the signs and symptoms of the disease are the result of what? A) Genetic dysfunction B) Upper and lower motor neuron lesions C) Decreased conduction of impulses in an upper motor neuron lesion D) A lower motor neuron lesion

D) A lower motor neuron lesion Myasthenia gravis is characterized by a weakness of muscles, especially in the face and throat, caused by a lower neuron lesion at the myoneural junction. It is not a genetic disorder. A combined upper and lower neuron lesion generally occurs as a result of spinal injuries. A lesion involving cranial nerves and their axons in the spinal cord would cause decreased conduction of impulses at an upper motor neuron.

The nurse caring for a patient in ICU diagnosed with Guillain-Barr syndrome should prioritize monitoring for what potential complication? A) Impaired skin integrity B) Cognitive deficits C) Hemorrhage D) Autonomic dysfunction

D) Autonomic dysfunction Based on the assessment data, potential complications that may develop include respiratory failure and autonomic dysfunction. Skin breakdown, decreased cognition, and hemorrhage are not complications of Guillain-Barr syndrome.

A middle-aged woman has sought care from her primary care provider and undergone diagnostic testing that has resulted in a diagnosis of MS. What sign or symptom is most likely to have prompted the woman to seek care? A) Cognitive declines B) Personality changes C) Contractures D) Difficulty in coordination

D) Difficulty in coordination The primary symptoms of MS most commonly reported are fatigue, depression, weakness, numbness, difficulty in coordination, loss of balance, spasticity, and pain. Cognitive changes and contractures usually occur later in the disease.

A patient with suspected Creutzfeldt-Jakob disease (CJD) is being admitted to the unit. The nurse would expect what diagnostic test to be ordered for this patient? A) Cerebral angiography B) ABG analysis C) CT D) EEG

D) EEG The EEG reveals a characteristic pattern over the duration of CJD. A CT scan may be used to rule out disorders that may mimic the symptoms of CJD. ABGs would not be necessary until the later stages of CJD; they would not be utilized as a diagnostic test. Cerebral angiography is not used to diagnose CJD.

The nurse is creating a plan of care for a patient who has a recent diagnosis of MS. Which of the following should the nurse include in the patients care plan? A) Encourage patient to void every hour. B) Order a low-residue diet. C) Provide total assistance with all ADLs. D) Instruct the patient on daily muscle stretching.

D) Instruct the patient on daily muscle stretching. A patient diagnosed with MS should be encouraged to increase the fiber in his or her diet and void 30 minutes after drinking to help train the bladder. The patient should participate in daily muscle stretching to help alleviate and relax muscle spasms.

A 73-year-old man comes to the clinic complaining of weakness and loss of sensation in his feet and legs. Assessment of the patient shows decreased reflexes bilaterally. Why would it be a challenge to diagnose a peripheral neuropathy in this patient? A) Older adults are often vague historians. B) The elderly have fewer peripheral nerves than younger adults. C) Many older adults are hesitant to admit that their body is changing. D) Many symptoms can be the result of normal aging process.

D) Many symptoms can be the result of normal aging process. The diagnosis of peripheral neuropathy in the geriatric population is challenging because many symptoms, such as decreased reflexes, can be associated with the normal aging process. In this scenario,the patient has come to the clinic seeking help for his problem; this does not indicate a desire on the part of the patient to withhold information from the health care giver. The normal aging process does not include a diminishing number of peripheral nerves.

A patient presents at the clinic complaining of pain and weakness in her hands. On assessment, the nurse notes diminished reflexes in the upper extremities bilaterally and bilateral loss of sensation. The nurse knows that these findings are indicative of what? A) Guillain-Barr syndrome B) Myasthenia gravis C) Trigeminal neuralgia D) Peripheral nerve disorder

D) Peripheral nerve disorder The major symptoms of peripheral nerve disorders are loss of sensation, muscle atrophy, weakness, diminished reflexes, pain, and paresthesia (numbness, tingling) of the extremities. Trigeminal neuralgia is a condition of the fifth cranial nerve that is characterized by paroxysms of pain in the area innervated by any of the three branches, but most commonly the second and third branches of the trigeminal nerve. Myasthenia gravis, an autoimmune disorder affecting the myoneural junction, is characterized by varying degrees of weakness of the voluntary muscles. Guillain-Barr syndrome is an autoimmune attack on the peripheral nerve myelin.

The nurse caring for a patient diagnosed with Guillain-Barr syndrome is planning care with regard to the clinical manifestations associated this syndrome. The nurses communication with the patient should reflect the possibility of what sign or symptom of the disease? A) Intermittent hearing loss B) Tinnitus C) Tongue enlargement D) Vocal paralysis

D) Vocal paralysis Guillain-Barr syndrome is a disorder of the vagus nerve. Clinical manifestations include vocal paralysis, dysphagia, and voice changes (temporary or permanent hoarseness). Hearing deficits, tinnitus, and tongue enlargement are not associated with the disease.

A patient diagnosed with Bells palsy is having decreased sensitivity to touch of the involved nerve. What should the nurse recommend to prevent atrophy of the muscles? A) Blowing up balloons B) Deliberately frowning C) Smiling repeatedly D) Whistling

D) Whistling Facial exercises, such as wrinkling the forehead, blowing out the cheeks, and whistling, may be performed with the aid of a mirror to prevent muscle atrophy. Blowing up balloons, frowning, and smiling are not considered facial exercises.

A nurse is teaching a client and her family about the diagnosis and treatment of Alzheimer's disease. Which of the following statements should the nurse identify as an indication that the family understands the teaching? A. "There is a test for Alzheimer's disease that can establish a reliable diagnosis" B. "The goal of medication therapy is to reverse the degenerative changes that can occur in brain tissue" C. "Early manifestations of Alzheimer's disease include mild tremors and muscular rigidity" D. "The medications that treat Alzheimer's disease can help delay cognitive changes"

D. "The medications that treat Alzheimer's disease can help delay cognitive changes."-Medications that treat Alzheimer's disease enhance the availability of acetylcholine, which can slow cognitive decline in some clients. "There is a test for Alzheimer's disease that can establish a reliable diagnosis."There is no specific test for identifying Alzheimer's disease, except direct examination of the brain on autopsy. Providers diagnose Alzheimer's disease based on manifestations and by ruling out other diseases. "The goal of medication therapy is to reverse the degenerative changes that can occur in brain tissue."None of the medications currently available reverse the course of Alzheimer's disease. "Early manifestations of Alzheimer's disease include mild tremors and muscular rigidity."Early manifestations include short-term memory loss, forgetfulness, and a shortened attention span. Mild tremors and muscular rigidity are manifestations of Parkinson's disease.

A nurse is caring for a client who has a basilar skull fracture following a fall from a ladder. Which of the following assessment findings should the nurse report to the provider? A. Glasgow Coma Scale score of 15 B. Intracranial pressure reading of 15 mm Hg C. Ecchymosis at base of skull D. Clear drainage from nose

D. Clear drainage from nose- Clear drainage from the nose indicates that cerebrospinal fluid is leaking from the skull fracture. This places the client at risk for meningeal infection because micro-organisms have access to the cranium through the nose and the tear in the dura. The nurse should report this finding to the provider. Glasgow Coma Scale score of 15- A Glasgow Coma Scale score of 15 indicates intact neurologic functioning and does not need to be reported to the provider. Intracranial pressure reading of 15 mm Hg-An intracranial pressure reading of 15 mm Hg is at the upper limit of the expected reference range and does not need to be reported to the provider. Ecchymosis at base of skull- A client who has a basilar skull fracture is likely to have ecchymosis at the base of the skull from a contusion and this finding does not need to be reported to the provider.

A nurse is performing a pain assessment for a client who is postoperative. Which of the following findings should the nurse use to determine the severity of the client's pain? A. Client's vital sign changes B. Client's report of the type of pain C. Client's nonverbal communication D. Client's report of pain on a pain scale

D. Client's report of pain on a pain scale- The nurse should use a client's report of pain on a standardized pain scale to determine the severity of the client's pain. Client's vital sign changes- A change in vital signs can identify that pain is present, but the nurse should use another finding to determine the severity of the client's pain. Client's report of the type of pain- report of the type of pain identifies the character of the pain, such as sharp or dull, but the nurse should use another finding to determine the severity of the client's pain. Client's nonverbal communication-Facial grimacing can identify that pain is present, but the nurse should use another finding to determine the severity of the client's pain.

A nurse is assessing a client who is quadriplegic following a cervical fracture at vertebral level C5. The client reports a throbbing headache and nausea. The nurse notes facial flushing and a blood pressure of 220/110mm Hg. Which of the following actions should the nurse take first? A. Administer hydralazine via IV bolus B. Loosen the client's clothing C. Empty the client's bladder D. Elevate the head of the client's bed

D. Elevate the head of the client's bed.- These assessment findings indicate that the client is experiencing autonomic dysreflexia and is at greatest risk for possible rupture of a cerebral vessel or increased intracranial pressure. The first action the nurse should take is to move the client from a supine to an upright position, which will result in rapid postural hypotension. Administer hydralazine via IV bolus.- The nurse should administer hydralazine, a potent vasodilator, to lower the client's blood pressure. However, there is another action the nurse should take first. Loosen the client's clothing.-The nurse should loosen the client's clothing because body temperature and tactile stimulation are triggers of autonomic dysreflexia. However, there is another action the nurse should take first. Empty the client's bladder.- The nurse should empty the client's bladder because a full bladder or a fecal impaction is a trigger of autonomic dysreflexia. However, there is another action the nurse should take first.

A nurse is caring for a client who has a history of status epilepticus and requires seizure precautions. Which of the following actions should the nurse take? A. Assess hourly for a spike in blood pressure B. Keep the client on bed rest C. Keep a padded tongue blade at the bedside D. Establish IV access

D. Establish IV access-The nurse should plan to establish IV access with a large-bore catheter and administer 0.9% sodium chloride if seizures are imminent. If the client is stable, the nurse should initiate a saline lock. Assess hourly for a spike in blood pressure.The nurse should check the client's vital signs and perform neurological checks after a seizure. However, a change in blood pressure does not correlate with an increased incidence of seizure activity. Keep the client on bed rest.A client who is at risk for seizures does not require bed rest. However, if seizures are imminent or frequent, the nurse should institute safety measures, such as placing the mattress on the floor or raising the side rails, according to agency policy. Keep a padded tongue blade at the bedside.MY ANSWERThe nurse should not plan to place objects, such as a padded tongue blade, in the client's mouth during a seizure because it can injure teeth and put the client at risk for aspirating tooth fragments. The tongue blade could also obstruct the client's airway.

While positioning a patient in bed with increased ICP, it important to avoid?* A. Midline positioning of the head B. Placing the HOB at 30-35 degrees C. Preventing flexion of the neck D. Flexion of the hips

D. Flexion of the hips The answer is D. Avoid flexing the hips because this can increase intra-abdominal/thoracic pressure, which will increase ICP.

A nurse is teaching an assistive personnel (AP) about providing care to a client following a total hip arthroplasty. Which of the following instructions should the nurse include? A. Avoid applying antiembolism stockings to the affected leg B. Have the client lean forward when moving from a sitting to a standing position C. Discourage the client from sitting in a wheelchair with the back reclined D. Place an abductor pillow between the client's legs when turning the client

D. Place an abductor pillow between the client's legs when turning the client.- The nurse should inform the AP that a client who had a total hip arthroplasty should maintain the hip in abduction following surgery to reduce the risk of dislocating the affected hip. The AP should place an abductor pillow between the client's legs when turning the client to keep the hips in abduction. Avoid applying antiembolism stockings to the affected leg.-The nurse should instruct the AP that a client who had a total hip arthroplasty should wear antiembolism stockings on both legs postoperatively to prevent the development of emboli in the lower extremities. Have the client lean forward when moving from a sitting to a standing position.-The nurse should instruct the AP that a client who had a total hip arthroplasty should use the unaffected leg and arms to push straight up to standing and not flex the affected hip more than 90°. Discourage the client from sitting in a wheelchair with the back reclined.- The nurse should instruct the AP that a client who had a total hip arthroplasty can sit in either an upright wheelchair or one with a back that reclines to prevent hip flexion greater than 90

A nurse is caring for a client who is recovering from a stroke an has right-sided homonymous hemianopsia. To help the client adapt to the hemianopsia, the nurse should take which of the following actions? A. Check the client's cheek on the affected side after meals to be sure no food remains there B. Encourage the client to sit upright with their head tilted slightly forward during meals C. Provide the client with eating utensils that have large handles D. Remind the client to look consciously at both sides of their meal tray

D. Remind the client to look consciously at both sides of their meal tray.- Clients who have right-sided homonymous hemianopsia have lost the right visual field of both eyes and might only eat the food they are able to see on the left half of the meal tray. Therefore, the nurse should remind the client to look at both sides of his meal tray to help compensate for the visual loss Check the client's cheek on the affected side after meals to be sure no food remains there.- Homonymous hemianopsia does not cause the client to pocket food. However, food can accumulate on the affected side of the mouth, so the nurse should place food on the unaffected side of the client's mouth when assisting with eating. Encourage the client to sit upright with their head tilted slightly forward during meals.- Homonymous hemianopsia does not cause dysphagia. However, as stroke can cause dysphagia, positioning the client upright and having them tilt their head forward to swallow can help prevent aspiration. Provide the client with eating utensils that have large handles.-Homonymous hemianopsia does not impair the client's fine motor skills. However, as stroke can impair fine motor skills, eating utensils that have a wide grip surface can help compensate for a weak hand grasp.

A nurse is planning to teach a client who has epilepsy and a new prescription for phenytoin. Which of the following instructions should the nurse plan to include? A. Rinse with antiseptic mouthwash instead of using dental floss B. Use an OTC antihistamine if a rash develops C. Slowly taper the medication after 6 consecutive months without seizure activity D. Take medications at a consistent time each day to maintain therapeutic blood levels

D. Take medications at a consistent time each day to maintain therapeutic blood levels.- The nurse should teach the client to take antiepileptic medications on a regular schedule to maintain therapeutic blood levels and achieve the maximum effect. Rinse with antiseptic mouthwash instead of using dental floss.The nurse should teach the client that phenytoin can cause gingival hyperplasia, an overgrowth of gum tissue. To minimize gum injury and discomfort, the client should brush and floss after each meal, massage their gums, and schedule dental examinations regularly. Use an over-the-counter antihistamine if a rash develops.The nurse should teach the client to stop taking phenytoin if a rash develops and to report the development of a rash to the provider immediately. An adverse effect of phenytoin therapy is the development of a measles-like rash. If left untreated, the rash could progress to Stevens-Johnson syndrome or toxic epidermal necrolysis. Slowly taper the medication after 6 consecutive months without seizure activity.The nurse should teach the client to continue taking antiepileptic medications even in the absence of seizures. Stopping the medication can lead to the return of seizures or the complication of status epilepticus.

During the eye assessment of a patient with increased ICP, you need to assess the oculocephalic reflex. If the patient has brain stem damage what response will you find?* A. The eyes will roll down as the head is moved side to side. B. The eyes will move in the opposite direction as the head is moved side to side. C. The eyes will roll back as the head is moved side to side. D. The eyes will be in a fixed mid-line position as the head is moved side to side.

D. The eyes will be in a fixed mid-line position as the head is moved side to side. The answer is D. This is known as a negative doll's eye and represents brain stem damage. It is a very bad sign.

The nurse is monitoring a client after supratentorial surgery. Which sign does the nurse report immediately to the provider?

Decorticate positioning The major complications of supratentorial surgery are increased intracranial pressure from cerebral edema or hydrocephalus and hemorrhage. Decorticate positioning indicates damage to the pathway between the brain and the spinal cord. The client usually is rigid with flexion of arms, clenched fists, and extended rigid legs. Periorbital edema and a small-to-moderate amount of serosanguineous drainage are expected after a craniotomy. Ecchymoses in the facial region, especially around the eyes, are expected after a craniotomy.

a nurse is preparing a client for an electroencephalogram (EEG). when the client asks the nurse what this test does which of the following responses should the nurse provide

EEG records the electrical activity to your brain. help provide info on seizure disorder, sleep disorder, inflammation, bleeding and migraine headaches

Which are risk factors for stroke?

High BP Previous stroke/TIA smoking use of oral contraceptives

A client has had a traumatic brain injury and is mechanically ventilated. Which technique does the nurse use to prevent increasing intracranial pressure (ICP)?

Maintaining PaCO2 levels at 35 mm Hg After the first 24 hours when a client is mechanically ventilated, keeping the PaCO2 levels at 35 mm Hg prevents vasodilation, which could increase ICP. CO2 is a powerful vasodilator. Turner's sign is a bluish gray discoloration in the flank region caused by acute pancreatitis. The head of the bed should be at 30 degrees; the Trendelenburg position will cause the client's ICP to increase. Although some suctioning is necessary, frequent suctioning should be avoided because it increases ICP.

The nurse is monitoring a postoperative craniotomy client with increased intracranial pressure (ICP). Which pharmacologic agent does the nurse expect to be requested to maintain the ICP within a specified range?

Mannitol (Osmitrol) Mannitol is an osmotic diuretic used specifically to treat cerebral edema. Glucocorticoids have no demonstrated benefit in reducing ICP. Hydrochlorothiazide is only a mild diuretic; a loop diuretic such as furosemide (Lasix) is commonly used along with mannitol to reduce ICP. Dilantin is used to treat seizure activity caused by increased ICP.

A client is admitted with a stroke (brain attack). Which tool does the nurse use to facilitate a focused neurologic assessment of the client?

National Institutes of Health Stroke Scale (NIHSS) Health care providers and nurses at primary stroke centers use a specialized stroke scale such as the NIHSS to assess clients. The MMSE is used primarily to differentiate among dementia, psychosis, and affective disorders. An intracranial pressure monitor would be requested by the health care specialist if signs and symptoms indicated increased intracranial pressure. A reflex hammer is used to assess deep tendon reflex response.

A client with a history of atrial fibrillation is receiving sodium heparin 24 hours after receiving thrombolytic therapy for a stroke. Which emergency drug does the nurse ensure is on the floor?

Protamine sulfate Protamine sulfate is used to reverse the effects of heparin in case of adverse effects. Narcan (naloxone) is used to reverse the effects of a narcotic overdose. Vitamin K is used to reverse the effects of warfarin. Physostigmine is an acetylcholinesterase inhibitor used to treat myasthenia gravis.

A client in the emergency department (ED) has slurred speech, confusion, and visual problems, and has been having intermittent episodes of worsening symptoms. The symptoms have a gradual onset. The client also has a history of hypertension and atherosclerosis. What does the nurse suspect that the client is probably experiencing?

Thrombotic stroke The client's symptoms fit the description of a thrombotic stroke. Symptoms of embolic stroke have a sudden onset, unlike this client's symptoms. The client would be in a coma if a hemorrhagic stroke had occurred. Intermittent episodes of slurred speech, confusion, and visual problems are transient ischemic attacks, which often are warning signs of an impending ischemic stroke.

a nurse in the emergency department is caring for a group of clients who all have an odor of alcohol on their breath and multiple injuries to the head and extremities. which of the following clients should the nurse assess first

a client who is difficult to arouse and is unable to respond to questions

A nurse is teaching a client who has a new diagnosis of simple partial seizures about auras. What statements by the client indicates an understanding of the teaching? a. an aura is a sensory warning that a seizure is imminent b. an aura is a continuous seizure in which seizures occur in rapid succession c. an aura is a period of sleepiness following a seizure d. an aura is a brief loss of consciousness accompanied by staring

a. an aura is a sensory warning that a seizure is imminent An aura is a sensory warning that a seizure is imminent. The aura can be similar to a hallucination and involve any of the senses. The client can report hearing bells, seeing lights, or smelling an odor.

A nurse is performing a neurologic assessment for a client who has a brain tumor. What finding should indicate to the nurse cranial nerve involvement? a. dysphagia b. positive babinski sign c. decreased deep-tendon reflexes d. ataxia

a. dysphagia Dysphagia, or difficulty swallowing, can occur as a result of damage to cranial nerves IX (glossopharyngeal) or X (vagus). A positive Babinski sign, or the turning up of the toes upon plantar stimulation, is associated with an upper motor neuron lesion. The cranial nerves primarily innervate the face, neck, and a few organs. Decreased deep-tendon reflexes indicate impairment in the electrical conduction of spinal nerves that interfere with reflex arcs. The cranial nerves primarily innervate the face, neck, and a few organs. Ataxia, or uncoordinated movements of the extremities, can indicate damage to the cerebellum or motor pathways. The cranial nerves primarily innervate the face, neck, and a few organs.

A nurse is providing teaching to the partner of a client who has a new diagnosis of Parkinson's disease about degenerative complications. The nurse should include in the teaching that what manifestations is the priority? a. dysphagia b. emotional lability c. impaired speech d. self-care dependency

a. dysphagia The nurse should apply the ABC priority-setting framework. This framework emphasizes the basic core of human functioning, which is having an open airway, being able to breathe in adequate amounts of oxygen, and circulating oxygen to the body's organs via the blood. An alteration in any of these can indicate a threat to life and should be the nurse's priority concern. When applying the ABC priority-setting framework, the airway is the priority because it must be clear and open for oxygen exchange to occur. Breathing is the second priority in the ABC priority-setting framework because adequate ventilatory effort is essential for oxygen exchange to occur. Circulation is the third priority in the ABC priority-setting framework because delivery of oxygen to critical organs only occurs if the heart and blood vessels are capable of efficiently carrying oxygen to them. Therefore, dysphagia is the priority manifestation because it can lead to aspiration.

A nurse is assessing a client who has a closed head injury and has received mannitol for manifestations of increased intracranial pressure. What findings should indicate to the nurse that the med is having a therapeutic effect? a. the client's serum osmolarity is 310 mOsm/L b. the client's pupils are dilated c. the client's heart rate is 56/min d. the client is restless

a. the client's serum osmolarity is 310 mOsm/L Mannitol is an osmotic diuretic used to reduce cerebral edema by drawing water out of the brain tissue. A serum osmolarity of 310 mOsm/L is desired. A decrease in cerebral edema should result in a decrease in ICP. All other options are manifestations of increased ICP

A nurse is assessing a client who has sustained a recent head injury. What finding should the nurse recognize as a manifestation of increased intracranial pressure? a. widened pulse pressure b. tachycardia c. periorbital edema d. decreased in urine output

a. widened pulse pressure A widening of the pulse pressure, the difference between the systolic and diastolic pressure, is a manifestation of increased intracranial pressure. Other manifestations include pupil changes, change in the level of consciousness, and nausea and vomiting.

a nurse is providing teaching to a client who has a new diagnosis of multiple sclerosis. the client asks the nurse about the usual course of MS. which of the following responses should the nurse make

acute episodes are usually followed by remissions, which can vary in duration

a nurse is caring for a client who has moderate Alzheimer's disease. which of the following actions should the nurse take

add gestures when speaking with the client

a nurse is teaching a client who has a new diagnosis of simple partial seizures about auras. which of the following statements by the client indicates an understanding of the teaching

an aura is a sensory warning that a seizure is imminent

a nurse ask a client to stand with her feet together and her eyes open. after a few seconds, the nurse asks the client to close her eyes. if the client begins to fall, the nurse should interpret this finding as a positive Romberg test, indicating which of the following alterations a. cerebellar dysfunction b. occipital lobe dysfunction c. increased intraocular d. macular degeneration

cerebellar dysfunction

a nurse is caring for a client who has received sedation. when the nurse applies nailbed pressure, the client withdraw his hand. the nurse should document this response as indicating a. confusion b. arousal c. orientation d. attention

arousal

a nurse is providing teaching to a client who has a new diagnosis of Meniere's disease. (disorder of the inner ear affecting balance and hearing). which of the following instruction should the nurse include in the teaching in the teaching? a. avoid bearing down b. increase caffeine intake c. avoid sudden movements d. increase sodium intake

avoid sudden movements

A nurse is providing discharge teaching to a client who is postop following cataract surgery and has an intraocular lens implant. What statement by the client indicates an understanding of the instructions? a. I will sleep on the affected side b. I will avoid bending over c. I will restrict my caffeine d. I will take aspirin to relieve my pain

b. I will avoid bending over The nurse should instruct the client to avoid activities that can increase intraocular pressure, such as lifting, bending, coughing, or performing the Valsalva maneuver. An increase in intraocular pressure can result in intraocular hemorrhage.

A nurse is assessing a client who is unconscious and has a rhythmical breathing pattern of rapid deep respirations, followed by rapid shallow respirations, alternating with periods of apnea. The nurse should document that the client is experiencing which of the following types of respirations? a. orthopnea b. cheyne-stokes c. paradoxical d. kussmaul

b. cheyne-stokes Cheyne-Stokes respirations is a breathing pattern of deep to shallow breaths, followed by periods of apnea. Cheyne-Stokes respirations can be the result of a drug overdose or increased intracranial pressure and can precede death.

A nurse is providing teaching to a client who has a new diagnosis of migraine headaches about interventions to reduce pain at the onset of a migraine. What instructions should the nurse include in the teaching? a. place a warm compress on your forehead b. darken the lights c. light a scented candle d. drink a caffeinated beverage

b. darken the lights The nurse should instruct the client to lie down in a dark room to reduce migraine pain. Use a cool cloth, avoid scents and caffeine

A nurse in a rehab center is performing an assessment for a client who is recovering from a left-hemisphere stroke. What finding should the nurse expect? a. reduced left-side motor function b. difficulty with speech c. impulsive behavior d. neglect of the left side of the body

b. difficulty with speech The left hemisphere of the brain is usually the dominant side and is responsible for language. This is always true for right-handed clients and for the majority of left-handed clients. Since this client is recovering from a left-hemisphere stroke, the nurse should anticipate that the client will have aphasia and require speech therapy to establish communication.

A nurse is reviewing the lab result of a lumbar puncture for a client who has manifestations of bacterial meningitis. What finding should the nurse expect? a. elevated glucose b. elevated protein c. presence of RBCs d. presence of D-dimer

b. elevated protein An LP is a diagnostic test in which cerebrospinal fluid is extracted for examination. Manifestations of bacterial meningitis include an increase of protein in the cerebrospinal fluid.

A nurse is providing discharge teaching to the family of a client who has a new diagnosis of a seizure disorder. The nurse should instruct the client's family to take what action first in the event of a seizure? a. reorient the client b. protect the client's head c. loosen constrictive clothing d. turn the client on his side

b. protect the client's head The nurse should apply the safety and risk reduction priority-setting framework. This framework assigns priority to the factor or situation posing the greatest safety risk to the client. When there are several risks to client safety, the one posing the greatest threat is the highest priority. The nurse should use Maslow's Hierarchy of Needs, the ABC priority-setting framework, or nursing knowledge to identify which risk poses the greatest threat to the client. The client is at greatest risk for injury from hitting his head; therefore, the first action is to protect the client's head from injury.

a nurse responds to a call from an assistive personnel that a client just had a seizure and is unconscious. which of the following assessments is the nurse's priority a. measure the client's vital signs b. perform a neurological examination c. check airway patency d. assess the client for injuries

check airway patency

A nurse is providing teaching to the family of a client who has a new diagnosis of amyotrophic lateral sclerosis (ALS). The nurse should include in the teaching that which of the following findings is an early manifestation of ALS? a. sensory dysfunction b. weakness of the distal extremities c. decreased vision d. aletered temp regulation

b. weakness of the distal extremities ALS is a progressive neurodegenerative disease that involves the motor nerve cells in the brain and the spinal cord causing muscle wasting, spasticity, and eventually paralysis. Early manifestations of ALS include increasing muscle weakness, especially involving the distal arms and legs (hands and feet), speech, swallowing, and breathing.

A nurse is providing discharge teaching to a client who is postop following scleral buckling to repair a detached retina. What instructions should the nurse include in the teaching? a. you can expect your vision to return immediately after the procedure b. you should avoid reading for 1 week c. you can remove eye shields when you are sleeping d. you should not lift objects that are more than 25 lbs.

b. you should avoid reading for 1 week The client should avoid reading and any activity that can cause rapid movement of the eye because of the risk for detachment of the retina.

a nurse is triaging clients during a mass casualty event. which of the following labels should the nurse assign to a client who has a head injury with fixed, dilated pupils a. red tag-- life threatening injuries b. yellow tag-major injuries not life threatening c. green tag--minor injuries/not life threatening or need immediate attention d. black tag--not expected to live and will die naturally

black tag--not expected to live and will die naturally

a nurse is preparing an older adult client who had a transient ischemic attack (TIA) for discharge. the nurse should teach the client to monitor which of the following parameters at home a. blood glucose b. blood pressure c. daily weight d. sensation in the feet

blood pressure

A nurse is assessing a client who is admitted to the facility for observation following a closed head injury. What is the priority assessment data the nurse should collect to determine a change in the client's neurologic status? a. vital signs b. body posture c. LOC d. exam of pupils

c. LOC The nurse should apply the urgent vs. nonurgent priority-setting framework. Using this framework, the nurse should consider urgent needs the priority because they pose more of a risk to the client. The nurse might also use Maslow's Hierarchy of Needs, the ABC priority-setting framework, or nursing knowledge to identify the most urgent finding. Therefore, the priority assessment is level of consciousness. A change in the client's level of consciousness can be the first indication of a change in neurologic status.

A nurse is providing teaching to a client who has a new diagnosis of Meniere's disease. What instructions should the nurse include in the teaching? a. avoid bearing down b. increase caffeine intake c. avoid sudden movements d. increase sodium intake

c. avoid sudden movements Ménière's disease is a disorder of the inner ear affecting balance and hearing, characterized by vertigo, hearing loss, and tinnitus. The nurse should instruct the client to avoid sudden movements that can increase manifestations.

A nurse in a clinic is providing teaching to an adolescent client who has recurrent external otitis. What instructions should the nurse include in the teaching? a. dry the ear canal with a cotton swab after swimming b. apply an ice pack to the ear to relieve pain c. instill a diluted alcohol solution into the ear after swimming d. irrigate the ear with cool tap water to clean

c. instill a diluted alcohol solution into the ear after swimming External otitis is an inflammation of the external auditory canal often due to the retention of water in the ear from swimming. After the inflammation is gone, the client can prevent recurrence of external otitis by instilling diluted alcohol drops to decrease bacteria and dry the external ear canal.

A nurse in an acute care facility is preparing to admit a client who has myasthenia gravis. What supplies should the nurse place at the client's bedside? a. metered-dose inhaler b. continuous passive motion machine c. oral-nasal suction equipment d. external defib pads

c. oral-nasal suction equipment The client who has myasthenia gravis is at risk for aspiration because of progressive weakness of the oropharyngeal muscles. Myasthenia gravis causes muscle weakness due to an autoimmune disease that affects the acetylcholine receptors. The nurse should place oxygen and oral-nasal suction equipment at the bedside in the event of aspiration or respiratory distress.

A nurse is caring for a client who is postop following a frontal craniotomy. The nurse should place the client in what position? a. trendelenburg b. prone c. semi-fowlers d. sims

c. semi-fowlers To prevent an increase in intracranial pressure, the nurse should position the client with his head midline and the head of the bed elevated 30º. This positioning permits blood flow to the client's brain while allowing venous drainage, thereby decreasing the postoperative risk of increased intracranial pressure.

A nurse is assessing a client who has a new diagnosis of acute angle-closure glaucoma. The nurse should anticipate the client to report which of the following manifestations? a. multiple floaters b. flashes of light in front of the eye c. severe eye pain d. double vision

c. severe eye pain Severe eye pain is a manifestation of acute angle-closure glaucoma. Other manifestations can include report of halos around lights, blurred vision, headache, brow pain, and nausea and vomiting.

A nurse is assessing a client who has Guillain-Barre syndrome. What finding should the nurse expect? a. tonic-clonic seizures b. report of a severe headache c. weakness in the lower extremities d. decreased LOC

c. weakness in the lower extremities Guillain-Barré syndrome, also called acute inflammatory demyelinating polyneuropathy, is an inflammatory disorder of the peripheral nerves. It is characterized by the rapid onset of ascending weakness and paralysis, starting at the lower extremities, and can advance to the upper extremities. Guillain-Barré syndrome is an inflammatory disorder of the peripheral nerves. All other choices are not characteristics of this disorder

A nurse is assessing a client who is postop following a craniotomy and has urine output of 600 mL/hr. The nurse suspects the client has manifestations of diabetes insipidus. What lab values should the nurse plan to obtain to assess DI? a. BUN b. Blood glucose c. Urine ketones d. Specific gravity

d. Specific gravity Diabetes insipidus is caused by damage to the hypothalamus or the pituitary gland as a result of cranial surgery, infection, or a tumor. It is a condition in which an inadequate amount of antidiuretic hormone is released and results in polyuria. A low specific gravity (1.001 to 1.003) is a manifestation of diabetes insipidus.

A nurse is providing teaching to a class about TIAs. What info should the nurse include in the teaching? a. can cause irreversible hemiparesis b. can be the result of cerebral bleeding c. can cause cerebral edema d. can precede an ischemic stroke

d. can precede an ischemic stroke TIAs are considered a manifestation of advanced atherosclerotic disease and often precede an ischemic stroke. Manifestations of a TIA include loss of vision in one eye, inability to speak, transient hemiparesis, vertigo, diplopia, numbness, and weakness. TIAs are brief episodes of a neurologic deficit that last less than 24 hr after onset without any permanent disabilities. TIAs are caused by a temporary reduction of oxygen supply to the brain, such as from a thromboembolism or cerebral vasospasm. A hemorrhagic stroke can be the result of cerebral bleeding. Cerebral edema can be the result of a stroke. TIAs do not produce edema of the cerebrum.

a nurse is providing teaching about degenerative complications to the partner of a client who has a new diagnosis of parkinson's disease. which of the following manifestation is the priority a. dysphagia b. emotional liability c. impaired speech d. self-care dependency

dysphagia

a nurse is caring for a client during the first 72 hr following cerebrovascular accident (CVA). which of the following actions should the nurse take

elevate the head of the bed 25 to 30 degree with the client in a neutral midline position

a nurse is reviewing the laboratory results of a lumber puncture (LP) for a client who has manifestations of bacterial meningitis. which of the following findings should the nurse expect a. elevated glucose b. elevated protein c. presence of RBCs d. presence of D-dimer

elevated protein

A nurse is teaching a client who has a new diagnosis of primary open-angle glaucoma (POAG). What information should the nurse include in the teaching? (select all) a. lost vision can improve with eye drops b. administer eye drops as needed for vision loss c. glasses will be necessary to correct the accompanying presbyopia d. driving can be dangerous due to the loss of peripheral vision e. laser surgery can help reestablish the flow of aqueous fluid

d. driving can be dangerous due to the loss of peripheral vision e. laser surgery can help reestablish the flow of aqueous fluid Lost vision can improve with eye drops is incorrect. Eye drops will not improve vision; however, they can reduce intraocular pressure and prevent further vision loss. Administer eye drops as needed for vision loss is incorrect. The client should administer eye drops on a regular schedule to reduce intraocular pressure. Glasses will be necessary to correct the accompanying presbyopia is incorrect. Presbyopia, which is a decrease in near vision that occurs after 40 years of age, is not related to POAG. Vision loss that occurs with POAG will not improve with glasses. Driving can be dangerous due to the loss of peripheral vision is correct. Damage to the optic nerve that occurs secondary to increased intraocular pressure causes a decrease in peripheral vision and can cause complete vision loss if not treated. Laser surgery can help reestablish the flow of aqueous humor is correct. Laser surgery can reopen the trabecular meshwork and widen the Canal of Schlemm.

A nurse is teaching a client who has myopia laser-assisted in situ keratomileusis surgery. The nurse should include in the teaching that what is an adverse effect of Lasik surgery? a. eyelid twitching b. photosensitivity c. introcular hemorrhage d. dry eyes

d. dry eyes LASIK surgery is a procedure that can correct nearsightedness, farsightedness, and astigmatism by changing the shape of the cornea. Adverse effects of LASIK surgery include dryness of the eyes and blurred vision.

A nurse is teaching a class of new parents about otitis media. What manifestations should the nurse include in the teaching? a. high-pitched sound heard in the ear b. intermittent rapid eye movement c. itching on the external canal d. feeling of fullness in the ear

d. feeling of fullness in the ear A client who has otitis media can develop a feeling of fullness in the ear. Other manifestations can include ear pain, a cracking sound when yawning or swallowing, and mild dizziness.

A nurse is providing teaching to the family of a client who has stage II Alzheimer's disease. What info should the nurse include in the teaching? a. place abstract pictures on the wall of the client's room b. provide music for the client using headphones c. reorient the client to reality frequently d. limit choices offered to the client

d. limit choices offered to the client Choices should be limited for the client who has stage II AD to reduce confusion and frustration.

A nurse is assessing a client who has a high-thoracic spinal cord injury. The nurse should identify what findings as a manifestation of autonomic dysreflexia? a. flushing of the lower extremities b. hypotension c. tachycardia d. report of a headache

d. report of a headache Autonomic dysreflexia is a neurologic emergency that can occur in clients who have a cervical or thoracic spinal cord injury above the level of T6. Autonomic dysreflexia can be triggered by a full bladder or distended rectum. Manifestations include a severe, throbbing headache; flushing of the face and neck; bradycardia; and extreme hypertension.

A nurse is reviewing the medical hx of a client who is scheduled for MRI exam of the c-spine. The nurse should alert the provider to what info in the client's hx that is a contraindication to the procedure? a. the client has a new tattoo b. the client is unable to sit upright c. the client has a hx of PVD d. the client has a pacemaker

d. the client has a pacemaker An MRI uses strong magnets and radio waves that are evaluated using computer technology to view three-dimensional images of the body. Since an MRI is magnetically generated, it is not indicated for use in the presence of certain medical implants. Clients who have cerebral aneurysm clips, cardiac pacemakers, or internal defibrillators cannot undergo an MRI because the strong magnetic force can interfere with these devices and obscure surrounding anatomical structures.

a nurse is providing teaching to a client who has a new diagnosis of migraine headaches about interventions to reduce pain at the onset of a migraine. which of the onset of the following instructions should the nurse include in the teaching a. place a warm compress on your forehead b. darken the lights c. light a scented d. drink a caffeinated

darken the lights

a nurse is assessing an older adult client for physiological changes that can occur with age. which of the following findings should the nurse expect? a. increase saliva production b. decrease sense of taste c. increase sense of smell d. decrease chest wall rigidity

decrease sense of taste

a nurse is preparing a client who has a brain tumor for computed tomography (CT). which of the following factors affects the manner in which the nurse will prepare the client for the scan

development of hives when eating shrimps

a nurse in a rehabilitation center is performing an assessment for a client who is recovering from a left-hemisphere stroke. which of the following findings should the nurse expect a. reduced left-sided motor function b. difficult with speech c. impulsive behavior d. neglect of the left side of the body

difficult with speech

a nurse is caring for a client who experienced a traumatic brain injury. which of the following findings indicates the client is experiencing increased intracranial pressure a. battle's sign b. periorbital edema c. dilated pupils d. halo sign

dilated pupils

a nurse is assessing a client who has a head injury with a possible skull fractures. which of the following finding should the nurse identify as an indication that the client might have a complication involving the eighth cranial nerve (CN VIII) a. dizziness and hearing loss b. weakness of a side of the tongue--CN XII c. facial droop and asymmetrical smile--CN VII d. loss of the same visual field in both eyes--CN II

dizziness and hearing loss

a nurse is teaching a client who has myopia about laser-assisted in situ keratomileusis (LASIK) surgery. which of the following is an adverse effect of LASIK surgery a. eyelid twitching b. photosensitivity c. intraocular hemorrhage d. dry eyes

dry eyes

a nurse is performing a neurological assessment for a client who has a brain tumor. which of the following findings should indicate cranial nerve involvement. a. dysphagia b. positive babinski sign c. decreased deep-tendon reflexes d. ataxia

dysphagia

A nurse is caring for a client who is experiencing autonomic dysreflexia due to a C5 spinal cord injury. after checking the client's vital signs, which of the following actions should the nurse perform next a. administer nifedipine b. place the client in a high-fowler's position c. check for urinary retention d. check for a fecal impaction

place the client in a high-fowler's position.

a nurse is providing discharge teaching to the family of a client who has a new diagnosis of a seizure disorder. the nurse should instruct the client's family to take which of the following actions first in the event of a seizure. a. reorient the client b. protect the client's head c. loosen constrictive clothing d. turn the client onto his side

protect the client's head

a nurse is caring for a client who has an impairment of cranial nerve II (visual challenges) which of the following actions should the nurse perform to promote the client's safety

provide an obstacle-free path for ambulation

an emergency room nurse is assessing a client who has a new traumatic brain injury. the nurse observes extension of the client's arms and legs, pronation of the arms, and plantar flexion of the feet. which of the following actions is the nurse priority

provide supplemental diuretic

a nurse is reviewing the medical history of s client who has presbyopia. with which of the following activities should the nurse expect the client to have difficulty.

reading the newspaper

during a neurological assessment, a nurse asks how the client arrived at the appointment and with whom. which of the following types of memory is the nurse testing a. remote b. immediate c. recall d. past

recall/recent

a nurse is caring for a client who has dementia and is experiencing anxiety. which of the following actions should the nurse take

redirect the client to a different activity with a small group of people

a nurse is providing discharge teaching to a client who has had a transient ischemic attack (TIA). which of the following instructions should the nurse include

reduce dietary sodium

during a neurological assessment, a nurse ask the client to name all of his children, their ages, and their birth dates. which of the following types of memory is the nurse testing? a. remote b. sensory c. immediate d. recall

remote

a nurse is assessing a client who has a high-thoracic spinal cord injury. the nurse should identify which of the following findings as a manifestation of autonomic dysreflexia a. flushing of the lower extremities b. hypotension c. tachycardia d. report of a headache

report of a headache

a nurse is caring for a client who has a brainstem injury. which of the following physiological functions should the nurse monitor a. understanding speech -temporal lobe b. respiratory effort-the medulla in the brainstem control the respiratory center c. decision-making ability-frontal lobe d. temperature control-hypothalamus

respiratory effort-the medulla in the brainstem control the respiratory center

a home health nurse is interviewing the adult child of a client who has alzheimer disease. the child is the client's sole caregiver and report feeling fatigued and overwhelmed. which of the following referrals should the nurse make for the caregiver a. attorney b. physical therapy c. respite care d occupational therapy

respite care

a nurse is assessing a client who report vision loss. the client describes the loss as beginning with a flash of light following a curtain across the field of vision. the nurse should identify that these manifestations indicates which of the following eye disorder a. glaucoma b. retinal detachment c. macular degeneration- decline of central vision d. cataracts-blurred vision/decrease color perception

retinal detachment-no pain

a nurse is caring for a client who has closed traumatic brain injury and is experiencing increased intracranial pressure (ICP). this increase an ICP is due to which of the following a. decreased cerebral perfusion b. leakage of cerebral spinal fluid c. rigid skull containing cranial contents d. brain herniated into the brainstem

rigid skull containing cranial contents

a nurse is reviewing the medical record of a client who is experiencing tinnitus in both ears. which of the following pieces of information in the client's medical record should the nurse identify as a risk factor for tinnitus a. use of hydrochlorothiazide b. chronic use of acetaminophen c. allergic external otitis d. sclerosis of the ossicles

sclerosis of the ossicles

a nurse is caring for a client who is postoperative following a frontal craniotomy. the nurse should place the client in which of the following position a. Trendelenburg b. prone c. semi-fowler's d. Sims

semi-fowler's


Related study sets

Chapter 15- Shampooing, Rinsing, & Conditioning

View Set

Respiratory Physiology (Lectures 1-7)

View Set